523 ophthalmology MCQs

20,374 views 79 slides Jan 02, 2021
Slide 1
Slide 1 of 79
Slide 1
1
Slide 2
2
Slide 3
3
Slide 4
4
Slide 5
5
Slide 6
6
Slide 7
7
Slide 8
8
Slide 9
9
Slide 10
10
Slide 11
11
Slide 12
12
Slide 13
13
Slide 14
14
Slide 15
15
Slide 16
16
Slide 17
17
Slide 18
18
Slide 19
19
Slide 20
20
Slide 21
21
Slide 22
22
Slide 23
23
Slide 24
24
Slide 25
25
Slide 26
26
Slide 27
27
Slide 28
28
Slide 29
29
Slide 30
30
Slide 31
31
Slide 32
32
Slide 33
33
Slide 34
34
Slide 35
35
Slide 36
36
Slide 37
37
Slide 38
38
Slide 39
39
Slide 40
40
Slide 41
41
Slide 42
42
Slide 43
43
Slide 44
44
Slide 45
45
Slide 46
46
Slide 47
47
Slide 48
48
Slide 49
49
Slide 50
50
Slide 51
51
Slide 52
52
Slide 53
53
Slide 54
54
Slide 55
55
Slide 56
56
Slide 57
57
Slide 58
58
Slide 59
59
Slide 60
60
Slide 61
61
Slide 62
62
Slide 63
63
Slide 64
64
Slide 65
65
Slide 66
66
Slide 67
67
Slide 68
68
Slide 69
69
Slide 70
70
Slide 71
71
Slide 72
72
Slide 73
73
Slide 74
74
Slide 75
75
Slide 76
76
Slide 77
77
Slide 78
78
Slide 79
79

About This Presentation

ophthalmology MCQs


Slide Content

1


MCQ - Ophthalmology

(523 questions)


Table of Contents
I - GENERAL MEDICINE (50 q)........................................................................................................................ 2
II - FUNDAMENTALS AND PRINCIPLES OF OPHTHALMOLOGY (53 q) .......................................................... 9
III - OPTICS, REFRACTION AND CONTACT LENS (35 q)................................................................................ 16
IV - OPHTHALMIC PATHOLOGY AND INTRAOCULAR TUMORS (45 q) ....................................................... 22
V - NEURO-OPHTHALMOLOGY (40 q) ......................................................................................................... 28
VI - PEDIATRIC OPHTHALMOLOGY AND STRABISMUS (50 q) .................................................................... 34
VII - ORBIT, EYELIDS AND LACRIMAL SYSTEM (45 q) ................................................................................. 42
VIII - EXTERNAL DISEASE AND CORNEA (43 q) ........................................................................................... 49
VIIII - INTRAOCULAR INFLAMMATION AND UVEITIS (50 q)....................................................................... 55
X – GLAUCOMA (34 q) ................................................................................................................................. 62
XI - LENS AND CATARACT (33 q) ................................................................................................................. 67
XII - RETINA AND VITREOUS (45 q) ............................................................................................................. 73

2
I - GENERAL MEDICINE (50 q)

1- Which one of the following bacteria evades phagocytosis in the
absence of antibody and complement because of polysaccharide
encapsulation?
A- Staphylococcus aureus
B- Staphylococcus epidermidis
C- Streptococcus pneumoniae
D- Pseudomonas aeruginosa

2- Which one of the following antibiotics is the treatment of choice for life-
threatening Staphylococcus aureus systemic infection?
A- Penicillin
B- Vancomycin
C- Gentamicin
D- A cephalosporin

3- Which one of the following antibiotics is the treatment of choice for
Streptococcus pyogenes infections (e.g., strep throat, impetigo)?
A- Penicillin
B- Vancomycin
C- Gentamicin
D- A cephalosporin

4- All except which one of the following is true about Lyme disease?
A- It is tick-borne.
B- Late manifestations are seen in the skin, joints, and nervous
system.
C - It is usually diagnosed by bacterial culturing.
D- The organism is sensitive to tetracycline.

5- Which one of the following statements is NOT correct about herpes
zoster?
A- Primary infection usually occurs in childhood in the form of
chickenpox (varicella).
B- No systemic antiviral treatment exists at this time.
C- Post-herpetic neuralgia may occur.
D- Herpes zoster occurs after a reactivation of a latent nerve infection.

6- Health care workers exposed to HIV may be given immediate treatment
with which one of the following medications?
A- Zidovudine (Retrovir)
B- Saquinavir (Invirase)
C- Ganciclovir (Cytovene)
D- HIV vaccine

7- β1-Adrenergic receptor site stimulation causes all except which one of
the following?
A- Vasoconstriction
B- Tachycardia
C- Increased myocardial contractility
D- Bronchoconstriction

3
8- The annual mortality rate of patients with asymptomatic carotid bruit has
been estimated at 4%. The most likely cause of death is which one of
the following?
A- Non-hemorrhagic stroke
B- Hemorrhagic stroke
C- Subarachnoid hemorrhage
D- Complications of heart disease

9- Which one of the following is most sensitive in distinguishing a
myocardial infarction from unstable angina or non-cardiac chest pain?
A- Elevation of the ST segment on the electrocardiogram
B- Echocardiogram
C- Elevated serum cardiac enzymes
D- Depression of the ST segment on exercise stress testing

10- All but which one of the following may be indicated for the
management of congestive heart failure?
A- α-Adrenergic antagonists
B- Digitalis
C- The calcium channel blocker diltiazem (Cardizem)
D- β-Adrenergic antagonists

11- Which one of the following does not increase serum high-density
lipoprotein cholesterol (HDL-C)?
A- Aerobic exercise
B- Moderate alcohol consumption
C- Gemfibrozil (Lopid)
D- Lovastatin (Mevacor)

12- Hypercholesterolemia is a risk factor for all but which one of the
following?
A- Arteritic ischemic optic neuropathy
B- Ischemic heart disease
C- Cerebrovascular disease
D- Peripheral vascular disease

13- Which one of the following is most indicative of restrictive pulmonary
disease?
A- Abnormal-appearing chest radiograph
B- FEV1 less than 80% predicted
C- Low PO2 in arterial blood gas
D- Total lung capacity less than 70% predicted

14- Which one of the following is the best test to monitor heparin therapy?
A- Prothrombin time (PT)
B- Partial thromboplastin time (PTT)
C- Bleeding time
D- Platelet count

15- Which one of the following is not used to dissolve existing clots?
A- Streptokinase
B- Urokinase
C- Heparin
D- Tissue plasminogen activator

4
16- Which one of the following statements is not correct about rheumatoid
arthritis?
A- It tends to affect the large joints.
B- Approximately 80% of patients are positive for rheumatoid factor.
C- Stiffness at rest often improves with use.
D- Extra-articular disease may be found.

17- A 50-year-old white male presents with acute nongranulomatous
anterior uveitis. He has had chronic back pain for years. Which one of
the following does NOT fit with his clinical syndrome?
A- Sacroiliitis on radiography
B- Spinal ankylosis on radiography
C- Positive HLA-DR4
D- Restrictive lung disease

18- Which one of the following is the most common ophthalmologic
manifestation of systemic lupus erythematosus?
A- Cranial nerve palsies
B- Retinal vascular disease
C- Cortical blindness
D- Sjögren’s syndrome

19- Renal disease is associated with all but which one of the following
diseases?
A- Rheumatoid arthritis
B- Scleroderma
C- Polyarteritis nodosa
D- Wegener granulomatosis

20- Which one of the following is most effective for treating Wegener
granulomatosis?
A- Aspirin
B- Nonsteroidal anti-inflammatory agents
C- Cyclophosphamide
D- Methotrexate

21- The definitive test for giant cell arteritis is which one of the following?
A- Wintrobe sedimentation rate
B- Westergren sedimentation rate
C- C-reactive protein
D- Temporal artery biopsy

22- In addition to uveitis, with or without hypopyon, which one of the
following is the most common ophthalmic manifestation of Behcet’s
syndrome?
A- Glaucoma
B- Retinal vasculitis
C- Corneal disease
D- Eyelid ulcers

5
23- Nonsteroidal anti-inflammatory agents include all but which one of the
following?
A- Aspirin
B- Ibuprofen (Advil, Motrin)
C- Cyclosporine
D- Rofecoxib (Vioxx)

24- Type 2 diabetes is characterized by all but which one of the following?
A- Genetic predisposition
B- Low basal insulin secretion early in the disease
C- Later age of onset than with type 1 diabetes
D- Increased visceral fat

25- The Diabetes Control and Complications Trial showed a decreased
risk of all but which one of the following?
A- Hypoglycemia
B- Development and progression of retinopathy
C- Development and progression of nephropathy
D- Development and progression of neuropathy

26- Which one of the following is not used to diagnose diabetes mellitus?
A- Random blood glucose exceeding 200 mg/dL
B- Glucose level exceeding 200 mg/dL during glucose tolerance test
C- Glycosylated hemoglobin (HbA1c)
D- Fasting serum glucose exceeding 126 mg/dL

28- Which one of the following choices is preferred for a diabetic patient
about to undergo major surgery?
A- Light breakfast with full regular insulin dose in early AM
B- Fasting with reduced insulin dose in AM
C- Fasting, withhold insulin, and in preoperative area start a dextrose
IV and give half of an insulin dose
D- Switch from insulin to oral hypoglycemic agent preoperatively

29- The most sensitive and specific test(s) for screening for thyroid
disease is which one of the following choices?
A- T3 level
B- Free T4 and sensitive TSH levels
C- Radioactive iodine uptake
D- Thyroid-binding globulin level

30- Which one of the following ocular structures is the most
radiosensitive?
A- Lens
B- Cornea
C- Retina
D- Optic nerve

6
31- Cancer is predominantly a genetic disease. Which one of the following
cancers may have a viral cause?
A- Breast
B- Colon
C- Lung
D- Cervical

32- Which one of the following statements is not correct about bipolar
disorder, formerly known as manic depression?
A- It is the most common form of depression.
B- There may be alternating periods of depression and elevated
mood.
C- It is treated with psychotherapy.
D- It is treated with lithium.

33- Which one of the following best describes the patient who claims to be
unable to work but has no organic basis for visual loss?
A- Conversion disorder
B- Hypochondriasis
C- Factitious disorder
D- Malingering

34- Fetal alcohol syndrome includes all but which one of the following?
A- Blepharophimosis
B- Cataract
C- Telecanthus
D- Optic nerve hypoplasia

35- The barbiturates have all but which one of the following effects?
A- Sedative
B- Hypnotic
C- Antidepressive
D- Anticonvulsive

36- Antidepressants include all but which one of the following?
A- Lithium
B- Monoamine oxidase inhibitors
C- Fluoxetine (Prozac)
D- Haloperidol (Haldol)

37- Parkinson disease is characterized by all but which one of the
following?
A- Increased rigidity
B- Excess dopamine production
C- Potential worsening of symptoms with neuroleptic drugs
D- Loss of neurons in the substantia nigra

7
38- Alzheimer disease is characterized by all but which one of the
following?
A- Neurofibrillary tangles
B- Progressive dementia in later life
C- A toxic cause in most cases
D- Extraneuronal amyloid plaques

39- Vaccines are currently available for all but which one of the following?
A- Rabies
B- Influenza types A and B
C- Meningococcus
D- Haemophilus influenzae

40- Which one of the following would argue against widespread screening
for a disease?
A- It is treatable or preventable.
B- It has a low prevalence.
C- It is generally asymptomatic.
D- The cost of the disease and its complications is high.

41- Which one of the following is not a risk factor for breast cancer?
A- Fibrocystic disease
B- First-degree relative with breast cancer
C- Early menarche
D- Nulliparity

43- Routine childhood vaccinations include all but which one of the
following?
A- HiB
B- Varivax
C- HepB
D- Influenza

44- Which one of the following should be immediately available during an
edrophonium (Tensilon) test?
A- Atropine sulfate
B- Epinephrine
C- Pyridostigmine (Mestinon)
D- Propranolol

45- An anesthesiologist would avoid which one of the following in a patient
with history of malignant hyperthermia?
A- Thiopental sodium
B- Nitrous oxide
C- Midazolam (Versed)
D- Succinylcholine

46- Which one of the following medications would be the most important
for a preoperative patient to take the morning of surgery?
A- Antihypertensive agent
B- Digoxin
C- Thyroid medication
D- Estrogen supplements

8
47- Which one of the following studies would best demonstrate the
incidence of a disease?
A- Case series
B- Case-control study
C- Cohort study
D- Randomized, prospective, controlled clinical trial

48- Advantages of interventional studies include all but which one of the
following?
A- Low cost
B- Low risk of bias
C- Best way to establish efficacy of treatment
D- Effective for establishing rates of complications of treatment

49- For populations with a normal distribution, the range of observations
with a mean ±3 standard deviations would include what percentage of
observations?
A- 68.3%
B- 95.5%
C- 99.7%
D- 100%

50- Which one of the following statistical tests would be best for studying
the effects of gender, smoking, cholesterol levels, blood pressure,
diabetes, and family history of heart disease on the incidence of heart
attack?
A- Student's t-test
B- Chi-square test
C- Analysis of variance
D- Multivariable analysis

9
II - FUNDAMENTALS AND PRINCIPLES OF OPHTHALMOLOGY (53 q)


1- Which ONE of the following extraocular muscles does NOT originate in the orbital apex?
A- Superior rectus
B- Inferior rectus
C- Superior oblique
D- Inferior oblique

2- Which ONE of the following nerve fibers synapses in the ciliary ganglion?
A- Sympathetic fibers of the carotid plexus
B- Parasympathetic fibers of cranial nerve III (oculomotor)
C- Motor fibers of cranial nerve III to the inferior oblique muscle
D- Sensory fibers of cranial nerve V (trigeminal)

3- Which one of the following rectus muscle tendons inserts closest to the corneal limbus?
A- Medial rectus
B- Lateral rectus
C- Superior rectus
D- Inferior rectus

4- The posterior lamella of the upper eyelid contain all EXCEPT which one of the following?
A- Levator aponeurosis
B- Müllers muscle
C- Orbicularis muscle
D- Conjunctival lining

5- The mucin-producing tissue or cells of the eye are which one of the following?
A- Goblet cells
B- Glands of Krause and Wolfring
C- Glands of Zeis and Moll
D- Meibomian glands

6- Which one of the following has the correct sequence of angle structures in the anterior
segment of the eye, proceeding from anterior to posterior?
A- Corneal endothelium, scleral spur, trabecular meshwork, Schwalbe's line, ciliary
recess
B- Ciliary recess, trabecular meshwork, Schwalbe's line, scleral spur, corneal
endothelium
C- Corneal endothelium, Schwalbe's line, trabecular meshwork, scleral spur, ciliary
recess
D- Ciliary recess, Schwalbe's line, trabecular meshwork, scleral spur, corneal
endothelium

7- Retinal pigment epithelial functions include all EXCEPT which one of the following?
A- Vitamin A metabolism
B- Formation of the basal lamina
C- Heat exchange
D- Conversion of 11-cis-retinaldehyde to all-trans-retinaldehyde

10

8- Transmission of neuronal impulses in the retina from photoreceptor rods and cones to the
optic nerve follows which one of the following sequences?
A- Outer nuclear layer, inner nuclear layer, ganglion cell layer, ganglion cell nerve fiber
layer
B- Ganglion cell nerve fiber layer, ganglion cell layer, inner nuclear layer, outer nuclear
layer
C- Inner nuclear layer, outer nuclear layer, ganglion cell layer, ganglion cell nerve fiber
layer
D- Outer nuclear layer, inner nuclear layer, ganglion cell nerve fiber layer, ganglion cell
layer

9- Which one of the following cranial nerves is NOT a true nerve?
A- Cranial nerve II (optic)
B- Cranial nerve III (oculomotor)
C- Cranial nerve IV (trochlear)
D- Cranial nerve V (trigeminal)

10- Which one of the following cranial nerves exits from the dorsal aspect of the midbrain?
A- Cranial nerve III (oculomotor)
B- Cranial nerve IV (trochlear)
C- Cranial nerve V (trigeminal)
D- Cranial nerve VI (abducens)

11- A lesion in the lower, posterior aspect of the left occipital lobe is most likely to affect which
quadrant of visual field in the right eye?
A- Upper nasal
B- Lower nasal
C- Upper temporal
D- Lower temporal

12- Aneurysms that affect the oculomotor nerve (cranial nerve III) commonly occur at the
junction of which two arteries?
A- Internal and external carotid arteries
B- Anterior cerebral and anterior communicating arteries
C- Basilar and posterior cerebral arteries
D- Posterior communicating and internal carotid arteries

13- Postganglionic parasympathetic fibers for reflex lacrimation are carried in which one of the
following cranial nerves?
A- Cranial nerve III (oculomotor)
B- Cranial nerve IV (trochlear)
C- Cranial nerve V (trigeminal)
D- Cranial nerve VI (abducens)

14- An expanding lesion in the cavernous sinus is LEAST likely to affect which one of the
following nerves?
A- Ophthalmic division of cranial nerve V (trigeminal)
B- Maxillary division of cranial nerve V
C- Mandibular division of cranial nerve V
D- Cranial nerve VI (abducens)

11

15- Reiger syndrome includes all EXCEPT which one of the following?
A- Oligodontia
B- Maxillary hypoplasia
C- Posterior keratoconus
D- Posterior embryotoxon

16- Which one of the following statements about colobomas is NOT correct?
A- Colobomas are inherited as an autosomal recessive trait.
B- Colobomas may involve the retina.
C- Colobomas may involve the iris.
D- Colobomas can create an absolute scotoma in a visual field.

17- Clinical findings in congenital rubella, acquired in the first trimester, may include all EXCEPT
which one of the following?
A- Deafness
B- Pigmentary retinopathy
C- Cataract
D- Facial maldevelopment

18- Bilateral optic nerve hypoplasia may occur in conjunction with which one of the following
clinical findings?
A- Absence of the septum pellucidum
B- Glaucoma
C- Cataract
D- Pigmentary retinopathy

19- Persistent hyperplastic primary vitreous is NOT associated with which one of the following?
A- Cataract
B- Peripheral retinal neovascularization
C- Microphthalmos
D- Retinal detachment

20- Which one of the following statements is NOT correct about oculodermal melanocytosis
(nevus of Ota)?
A- It is occasionally associated with glaucoma.
B- Hyperpigmentation may involve the uveal tract.
C- It is usually unilateral.
D- There is no risk of malignant melanoma.

21- Which one of the following gene groups is a homeotic gene group that appears to regulate
activity or expression of genes guiding embryonic development of cells into body parts?
A- PAX
B- RET
C- GLC
D- BAX

22- Which one of the following statements about the anti-oncogene p53 is correct?
A- It codes for a protein with tumor antiangiogenesis effect.
B- It prevents cells from proliferating if their DNA is irreparably damaged.
C- It codes for a protein with direct anti-tumor effect.
D- It stimulates tumorigenesis.

12

23- Which one of the following disorders is NOT associated with a defect in a mitochondrial
gene?
A- Leber hereditary optic neuropathy
B- Chronic progressive external ophthalmoplegia
C- Neuropathy, ataxia, and retinitis pigmentosa (NARP)
D- Retinoblastoma

24- Which one of the following is an indication for chromosomal analysis?
A- Clinical diagnosis in newborns affected by multisystem malformations
B- Multiple miscarriages
C- Studies of malignancies
D- All of the above

25- Which one of the following is NOT correct about retinoblastoma?
A- It may be associated with chromosome 13 long arm (13q14) deletion syndrome.
B- Incidence of about 1 in 20,000 live births
C- About 50% of patients with hereditary retinoblastoma have a family history of the
disease.
D- The hereditary pattern of familial retinoblastoma is autosomal dominant.

26- Tay-Sachs disease is associated with all EXCEPT which one of the following?
A- Autosomal dominant inheritance
B- Macular cherry-red spot
C- Optic atrophy
D- A defect in the enzyme hexosaminidase

27- Homocystinuria is associated with which one of the following ocular findings?
A- Glaucoma
B- Cataract
C- Dislocated lens
D- Retinitis pigmentosa

28- Which one of the following statements is NOT correct about autosomal recessive
inheritance?
A- The mutant gene usually causes a milder form of the disease in the heterozygote.
B- The ratio of normal to affected in siblings is 3:1.
C- Both genders are equally affected.
D- Affected individuals may have children who are asymptomatic carriers of the gene.

29- A certain ocular disorder is found to occur more often in males than females, but affected
males do not transmit the disorder. However, virtually every son and daughter of an affected
female inherits the trait. What is the most likely mode of inheritance of this disorder?
A- Autosomal dominant
B- Autosomal recessive
C- X-linked recessive
D- Mitochondrial

30 Which is associated with a missing chromosome?
A- Trisomy 13 (Patau) syndrome
B- Trisomy 21 (Down) syndrome
C- Turner syndrome
D- Klinefelter syndrome

13

31- Which one of the following can be a mutagen?
A- Chemicals
B- Radiation
C- Viruses
D- All of the above

32- Which one of the following immunoglobulins is found in the tear film?
A- IgA
B- IgM
C- IgE
D- IgD

33- Which one of the following is found in a significantly higher concentration in the aqueous
humor than in blood?
A- Sodium (Na+)
B- Potassium (K+)
C- Bicarbonate (HCO3
-
)
D- Ascorbic acid (vitamin C)

34- Which one of the following is an α1-adrenergic agonist?
A- Phenylephrine (Neo-Synephrine)
B- Apraclonidine (Iopidine)
C- Betaxolol (Betoptic)
D- Thymoxamine

35- The blood–aqueous barrier has the greatest effect on aqueous concentration compared with
blood concentration of which one of the following?
A- Protein
B- Ascorbic acid (vitamin C)
C- Bicarbonate
D- Glucose

36- Which sugar has been implicated in cataractogenesis in diabetics?
A- Galactose
B- Glucose
C- Sorbitol
D- Mannitol

37- Which one of the following is the largest constituent of vitreous?
A- Collagen
B- Hyaluronic acid
C- Sodium
D- Water

38- Photoreceptor cones have one of three visual pigments with absorptive maxima of 570 nm,
540 nm, and 440 nm. Which one of these would absorb blue light the most?
A- 570 nm
B- 540 nm
C- 440 nm
D- None of the above

14

39- Cone density is greatest in which area of the retina?
A- Macula
B- Peripapillary region
C- Arcuate regions
D- Peripheral retina

40- Which vitamin is most critical for photoreceptor response to light?
A- A
B- B
C- C
D- D

41- Functions of the retinal pigment epithelium (RPE) include all EXCEPT which one of the
following?
A- Maintenance of retinal adhesion
B- Development of photoreceptors during embryogenesis
C- Transport of metabolites to and from the retina
D- Transmission of light-generated neuronal impulses from photoreceptors to the
ganglion cells

42- Methods of increasing drug penetration through the normal cornea include all EXCEPT
which one of the following?
A- Increasing drug concentration
B- Reducing drug lipid solubility
C- Adding a surfactant (benzalkonium chloride)
D- Adding a high-viscosity agent (methylcellulose, polyvinyl alcohol)

43- Clinical effects of direct-acting muscarinic agents (miotics) include all EXCEPT which one of
the following?
A- Increased myopia
B- Decreased range of accommodation
C- Central anterior chamber shallowing
D- Reduced night vision

44- Which one of the following is a direct-acting cholinergic agent?
A- Pilocarpine (Isoptocarpine, Pilocar, Pilostat)
B- Echothiophate iodide (Phospholine iodide)
C- Physostigmine (Eserine)
D- Demecarium bromide (Humorsol)

45- Which one of the following statements is CORRECT about cocaine?
A- It directly stimulates adrenergic receptors.
B- It directly stimulates muscarinic receptors.
C- It blocks re-uptake of norepinephrine.
D- It blocks adrenergic receptors.

46- Dapiprazole hydrochloride (Rev-Eyes) most effectively reverses the effect of which one of
the following agents?
A- Tropicamide (Mydriacyl)
B- Cyclopentolate (Cyclogyl)
C- Scopolamine (Hyoscine)
D- Phenylephrine (Neo-Synephrine)

15

47- Which one of the following antiglaucoma agents is preferred in moderate to severe
congestive heart failure?
A- Timolol (Timoptic, Betimol)
B- Betaxolol (Betoptic)
C- Carteolol (Ocupress)
D- None of the above

48- Which one of the following antiglaucoma agents is preferred in mild asthma?
A- Timolol (Timoptic, Betimol)
B- Betaxolol (Betoptic)
C- Carteolol (Ocupress)
D- None of the above

49- Systemic side effects of oral carbonic anhydrase inhibitors include all EXCEPT which one of
the following?
A- Paresthesias
B- Weight loss
C- Hyperkalemia
D- Metabolic alkalosis

50- Which one of the following is NOT a mast cell stabilizer or antihistamine?
A- Lodoxamide (Alomide)
B- Cromolyn sodium (Crolom)
C- Olopatadine hydrochloride (Patanol)
D- Ketorolac tromethamine (Acular)

51- Compared with first-generation cephalosporins, third-generation cephalosporins have
enhanced activity against all but which of the following organisms?
A- Gram-positive cocci
B- Gram-negative bacilli
C- Haemophilus influenzae
D- Pseudomonas aeruginosa

52- Antiviral agents include all EXCEPT which one of the following?
A- Trifluridine (Viroptic)
B- Amphotericin-B
C- Vidarabine (ara-A)
D- Idoxuridine (Stoxil)

53- Which one of the following local/regional anesthetic agents has the longest duration of
effect?
A- Lidocaine
B- Tetracaine
C- Bupivacaine
D- Mepivicaine

16
III - OPTICS, REFRACTION AND CONTACT LENS (35 q)

1- An object is placed 1 m in front of a –1 D spherical lens. The –1 D lens, in turn, is positioned
1.5 m in front of a +1.5 D spherical lens. Where does the –1 D lens form an intermediate
image?
A- At optical infinity
B- 2 m in front of the lens
C- 1 m in front of the lens
D- 0.5 m in front of the lens
E- 2 m behind the lens

2- An object is placed 1 m in front of a –1 D spherical lens. The –1 D lens, in turn, is positioned
1.5 m in front of a +1.5 D spherical lens. Describe the intermediate image.
A- Upright, real, magnified
B- Upright, real, minified
C- Upright, virtual, magnified
D- Upright, virtual, minified
E- Inverted, virtual, minified

3- An object is placed 1 m in front of a –1 D spherical lens. The –1 D lens, in turn, is positioned
1.5 m in front of a +1.5 D spherical lens. What is the size of the intermediate image as
compared to the object?
A- Indeterminate
B- One fourth the size
C- Half the size
D- Same size
E- Twice the size

4- An object is placed 1 m in front of a –1 D spherical lens. The –1 D lens, in turn, is positioned
1.5 m in front of a +1.5 D spherical lens. What is the location of the final image?
A- 1 m in front of the second lens
B- 1 m behind the second lens
C- 4 m behind the second lens
D- 10 m behind the second lens
E- At optical infinity

5- An object is placed 1 m in front of a –1 D spherical lens. The –1 D lens , in turn, is positioned
1.5 m in front of a +1.5 D spherical lens. Describe the final image.
A- Upright, real, magnified
B- Upright, real, minified
C- Inverted, real, magnified
D- Inverted, real, minified
E- Inverted, virtual, minified

6- An object is placed 1 m in front of a –1 D spherical lens. The –1 D lens, in turn, is positioned
1.5 m in front of a +1.5 D spherical lens. What is the size of the final image as compared to
the object?
A- Indeterminate
B- One fourth the size
C- Half the size
D- Same size
E- Twice the size

17
7- Complete the sentence: The ability of a light wave from a laser to form interference fringes
with another wave from the same beam, separated in time, is a measure of its
A- temporal coherence.
B- spatial coherence.
C- polarization.
D- directionality.
E- intensity.

8- How much does a 15∆ prism bend light, in degrees?
A- 7.51°
B- 8.53°
C- 12.4°
D- 15°
E- 86.2°

9- The anterior and posterior focal points of a thin lens are located at different distances from
the lens. Additionally, the nodal point of the lens does not correspond with the anatomic
center of the lens. Which of the following is TRUE?
A- This is not possible.
B- The lens is a spherocylinder.
C- The optical characteristics described are only seen in thick lens or multi-element lens
systems.
D- Media of different refractive indices bound the lens.
E- Two separated principal planes must be used to define the lens mathematically.

10- Complete the sentence: The far point of the non-accommodated myopic eye
A- and the fovea are corresponding points.
B- is posterior to the eye, optically speaking.
C- is nearer to the eye than the point of focus of the fully accommodated eye.
D- cannot be moved by placing a lens in front of the eye.

11- Complete the sentence: The near point of the fully accommodated hyperopic eye
A- is beyond infinity, optically speaking.
B- is between infinity and the cornea.
C- is behind the eye.
D- is beyond minus infinity, optically speaking.
E- cannot be determined without additional information.

12- You are about to write the postoperative spectacle prescription for a cataract surgery patient
with macular degeneration. The best choice for a reading add for the patient with 20/70 best-
corrected vision is
A- +3.00 D
B- +3.50 D
C- +7.00 D
D- A 3.5x magnifier

13- Complete the sentence: Proper distance visual acuity testing for a low-vision patient
includes all EXCEPT
A- a testing chart with symbols arranged in rows of decreasing size that are equally
legible.
B- non-standardized room illumination.
C- a Snellen visual acuity chart.
D- a +1.00 D lens placed over the patient’s distance refraction.

18
14- Complete the sentence: A patient with moderately low vision (20/160 in each eye) wishes a
prescription to be able to read; the best choice would be
A- +8.00 D single-vision reading spectacle.
B- +4.00 D half-glass reader with a total of 6∆ BI prism.
C- +8.00 D half-glass reader with a total of 10∆ BI prism.
D- +8.00 D half-glass reader with 10∆ BI prism per lens.
E- an 8.0x magnifier.

15- Complete the sentence: Proper medical management of a patient with macular
degeneration and recent visual deterioration to the “legal blindness” level (20/200) should
include
A- a 10x magnifier for reading.
B- referral to an orientation and mobility specialist.
C- a spectacle prescription for prismatic half-eye glasses for reading.
D- all of the above.
E- none of the above.

16- Appropriate testing and recording of near visual acuity for a low-vision patient includes
(choose from among the following):
1. Standard reading distance of 25 cm
2. Non-standardized illumination
3. Nomogram utilizing reading distance (in meters) and letter size (in Jaeger units)
4. Use of a handheld card
A- 4 alone
B- 1 and 3
C- 2 and 4
D- All of the above
E- None of the above

17- A patient with congenital nystagmus has a null point measured to be 10
o
to the left of
fixation. The appropriate prism prescription to rectify the induced head turn is
A- 10∆ BI OS, 10BO OD
B- 10∆ BI OD, 10∆ BO OS
C- 20∆ BI OS, 20∆ BI OD
D- 20∆ BI OD, 20∆ BO OS
E- 20∆ BI OS, 20∆ BO OD

18- What is the retinal magnification of an eye with a refractive error of +5 D when viewed with a
direct ophthalmoscope?
A- 13.75x
B- 15.00x
C- 10.75x
D- 5.00x

19- Using a keratometer, you determine the radius of curvature of the central cornea to be 9.0
mm. Assuming an “averaged” corneal refractive index of 1.3375, estimate the refractive
power of the central cornea.
A- 37.50 D
B- 1.33 D
C- 7.00 D
D- 27.00 D

19
20- If a cornea has an anterior radius of curvature of 7.7 mm, a posterior radius of curvature of
6.8 mm, and a center thickness of 0.5 mm, what will its dioptric power be if it is submerged in
water?
A- 0 D
B- –0.29 D
C- +2.47 D
D- +14.3 D
E- +37.6 D

21- Complete the sentence: The power of an intraocular lens should be increased
A- as the power of the cornea increases and the axial length increases.
B- as the power of the cornea decreases and the axial length increases.
C- as the power of the cornea increases and the axial length decreases.
D- as the power of the cornea decreases and the axial length decreases.

22- Which one of the following laser properties is not clinically important in ophthalmic
applications?
A- Energy level
B- Power level
C- Pulse duration
D- Polarity
E- Focal spot size

23- Complete the sentence: Multifocal IOLs
A- offer increased image clarity and contrast for both near and far viewing.
B- are independent of pupil size if they are well centered.
C- offer a trade-off between decreased image quality and increased depth of focus.
D- [are indicated for patients who do not wish to minimize the use of spectacles.

24- Complete the sentence: Considered as a whole, the focal planes of a Keplerian telescope
are located
A- at the objective lens.
B- at the ocular lens (eyepiece).
C- between the ocular and the objective.
D- the planes do not exist.

25- Complete the sentence: The SRK formula does not specifically require a measurement of
anterior chamber depth because
A- the formula was specifically designed to eliminate the need for this measurement.
B- regression analysis did not show increased accuracy when anterior chamber depth
was included in the IOL formula.
C- modern IOLs are all at about the same anterior chamber depth.
D- the postoperative anterior chamber depth is not necessarily the same as the
measured preoperative anterior chamber depth.

26- The spherocylindrical transposition of the combination of two cylinders: +1.75 x 150
o
is
which of the following?
A- +1.75 with –4.00 x 060
o

B- +4.00 with –1.75 x 150
o

C- –2.25 with +4.00 x 060
o

D- +2.25 with –4.00 x 150
o

E- –4.00 with +1.75 x 150
o

20
27- A myope’s glasses are prescribed incorrectly and he is overminused by 1 D. When he
wears them, his near point of accommodation is 20 cm. What is his refractive error and
amplitude of accommodation?
A- There is not enough information to solve this problem.
B- –1 D, 6 D
C- –5 D, 6 D
D- –6 D, 6 D
E- –6 D, 5 D

28- Complete the sentence: In bifocal design, image jump may be minimized by
A- placing the optical center of the segment as close as possible to the top of the
segment.
B- placing the top of the segment as close a possible to the distance optical center.
C- using a smaller bifocal segment.
D- using a blended bifocal segment having no visible line of separation.
E- lowering the bifocal segments by 3 mm.

29- An angle of 45
o
corresponds to how many prism diopters?
A- 45
B- 22.5
C- 90
D- 100

30- A patient with a comitant 3∆ right hyperphoria wears the following spectacle correction: OD
+3.00 + 1.0 x 180
o
, OS -1.00 + 0.50 x 090
o
; What is the vertical imbalance in the reading
position, 8 mm below the optical centers of the lenses?
A- 1∆ right hyperphoria
B- 1∆ left hyperphoria
C- 4∆ right hyperdeviation
D- 5∆ right hyperdeviation
E- 7∆ right hyperdeviation

31- Complete the sentence: In the derivation of the IOL formula using geometrical optics
A- the index of refraction of the IOL is ignored because it does not differ significantly from
that of the aqueous and vitreous.
B- the refractive contribution of the cornea may be neglected because the light reaches
the IOL plane after it has already passed though the corneal surface.
C- the formula for change of vergence with change in location must be modified because
of the index of refraction of the aqueous.
D- the anterior chamber depth may be neglected, since studies have shown a negligible
increase in accuracy if it is included in power calculations.

33- Complete the sentence: A child has a cycloplegic refraction of OD +5.00, OS +1.00; this
anisometropia should be
A- fully corrected.
B- partially corrected.
C- left uncorrected.
D- treated by occlusion.
E- treated by pleoptics.

21
34- Complete the sentence: When prescribing bifocal lenses for a myope
A- the practitioner should leave the choice of the segment type to the optician.
B- a round-top segment is preferred because of its thin upper edge, which causes less
prismatic effect.
C- a flat-top segment is preferred because it lessens image jump.
D- the one-piece shape is indicated for adds greater than +2.00 D.
E- a split bifocal should be used because myopes do not accept bifocals easily.

35- If K measurements are 7.90 (horizontal) / 7.70 (vertical) and the refractive error is –3.25 +
0.50 axis 90
o
and the base curve of the RGP contact lens is chosen to be 7.80 mm, what is
the anticipated (1) power needed by the contact lens and (2) amount of the residual
astigmatism?
A- (1) –3.25 sphere and (2) –0.50 axis 90
o

B- (1) –3.75 sphere and (2) –0.50 axis 90
o

C- (1) –3.25 sphere and (2) –0.50 axis 180
o

D- (1) –3.75 sphere and (2) –0.50 axis 180
o

22
IV - OPHTHALMIC PATHOLOGY AND INTRAOCULAR TUMORS (45 q)

1- Which of the following choices best describes the sequence of initiation of specific processes
during wound healing in clear cornea?
A- Epithelial wound repair, wound edge digestion, fibroblast activation, wound contraction
B- Wound edge digestion, epithelial wound repair, fibroblast activation, wound
contraction
C- Fibroblast activation, wound contraction, wound edge digestion, epithelial wound
repair
D- Wound edge digestion, epithelial wound repair, fibroblast activation, wound
contraction

2- Which of the following special techniques in diagnostic pathology is quantitative in its
standard form?
A- Immunohistochemistry
B- Electron microscopy
C- Flow cytometry
D- PCR-based gene rearrangement studies

3- Which of the following best describes how macular dystrophy of the cornea differs from
granular and lattice dystrophies?
A- Deposited material and visual disability
B- Genetic transmission, deposited material, and visual disability
C- Genetic transmission, tissue distribution, visual disability, and deposited material
D- Genetic transmission and tissue distribution

4- Which of the following would not be detected by Prussian blue staining?
A- Fleischer line
B- Stocker line
C- Ferry line
D- Krukenberg spindle

5- Which is not a histologic pattern of orbital rhabdomyosarcoma in the current classification?
A- Embryonal
B- Pleomorphic
C- Alveolar
D- Differentiated

6- In adults, the most common intraocular malignant tumor and most common site of
involvement are
A- Melanoma, choroid
B- Metastasis, iris
C- Melanoma, iris
D- Metastasis, choroid

7- Two tumors commonly associated with so-called masquerade syndromes are
A- Conjunctival lymphoma, choroidal melanoma
B- Conjunctival lymphoma, intraocular lymphoma
C- Eyelid sebaceous carcinoma, intraocular lymphoma
D- Basal cell carcinoma, retinoblastoma

23

8- The most common secondary tumors in retinoblastoma patients within and outside of the
field of ocular radiation are
A- Within, fibrosarcoma; outside, osteosarcoma
B- Within, osteosarcoma; outside, melanoma
C- Within, osteosarcoma; outside, pinealoblastoma
D- Within, osteosarcoma; outside, osteosarcoma

9- A family history of retinoblastoma is present in what percent of newly diagnosed
retinoblastoma patients?
A- 1%
B- 6%
C- 18%
D- 40%

10- Which of the following statements about pleomorphic adenoma of the lacrimal gland is
FALSE?
A- It can recur in a diffuse manner.
B- It can transform to a malignant tumor if present long enough.
C- Recurrences can transform to malignancy.
D- It can resolve spontaneously.

11- Which best reflects in increasing order of the age of presentation of retinoblastoma patients
with the following retinoblastoma history?
A- Family history, unilateral, bilateral
B- Unilateral, bilateral, family history
C- Unilateral, bilateral, family history
D- Family history, bilateral, unilateral

12- The modified Callender classification of ocular melanoma applies to melanomas of the
A- Eyelid
B- Conjunctiva
C- Iris
D- Choroid

13- Which of the following ocular histologic changes is NOT considered to be associated with
diabetes mellitus?
A- Lacy vacuolization of the iris
B- Retinal hemorrhages
C- Iris hemorrhages
D- Thickened basement membranes

14- Histologic differentiation between primary and recurrent pterygia can be based on
A- The degree of vascularity in the lesion
B- The presence of more fibrous tissue in the recurrent lesion
C- The absence of elastotic degeneration in recurrent pterygia
D- The presence of Bowman’s layer in recurrent but not primary pterygia

15- Which of the following is NOT a point of firm attachment between the sclera and uvea?
A- Ora serrata
B- Scleral spur
C- Internal ostia of vortex veins
D- Peripapillary tissue

24

16- Which of the following extraocular muscles inserts farthest from the limbus?
A- Superior rectus
B- Inferior rectus
C- Inferior oblique
D- Superior oblique

17- Which of the following antigens would allow identification of a spindle cell tumor as a
melanoma?
A- Cytokeratin
B- S-100
C- HMB-45
D- Desmin

18- Histologically, the term angle recession refers to which of the following conditions?
A- A tear between the ciliary body and the sclera
B- A tear between the iris and ciliary body
C- A tear between the longitudinal and circular portions of the ciliary muscle
D- Posterior displacement of the iris root without alteration of the ciliary body

19- Which of the following is NOT a chromosomal locus known to be associated with Axenfeld-
Reiger syndrome?
A- 15q5
B- 4q25
C- 13q14
D- 6p25

20- The sclera shows what variation in thickness and is composed primarily of what type of
collagen?
A- Tenfold thickness variation and type I collagen
B- Threefold thickness variation and type IV collagen
C- Twofold thickness variation and type IV collagen
D- Threefold thickness variation and type I collagen

21- Anterior scleritis is likely to
A- Be bilateral in 50% of cases
B- Be associated with pain on eye movement
C- Cause exophthalmos
D- Be painless

22- Mutations in the fibrillin gene on chromosome 15 lead to lens displacement
A- Down
B- Inferonasally
C- Superotemporally
D- Anteriorly

23- Which of the following is the most common primary malignancy of the eyelid?
A- Basal cell carcinoma
B- Squamous cell carcinoma
C- Sebaceous carcinoma
D- Melanoma

25


24- When a parent has bilateral retinoblastoma, each child
A- Has an 85% chance of developing retinoblastoma
B- Who develops retinoblastoma will have bilateral disease
C- Will be affected only if male
D- Has a 45% chance of developing retinoblastoma

25- In the final scar of the average wound,
A- Myofibroblasts and fibroblasts contract
B- Most of the vessels persist
C- The recently laid collagen parallels the uninjured surrounding collagen
D- Macrophages remain, producing enzymes

26- When enucleated, most blind, painful eyes have either
A- Absolute glaucoma or expulsive hemorrhage
B- Phthisis bulbi or absolute glaucoma
C- Fibrous ingrowth or absolute glaucoma
D- Epithelial downgrowth or absolute glaucoma

27- Conjunctival and corneal epithelia
A- Never grow inside the eye
B- Can invade the eye by burrowing through intact cornea
C- Produce free-floating solid masses of cells in the eye
D- May grow down a tract or wound edge to the inside of the eye

28- Cotton-wool spots: which one of the following statements is CORRECT?
A- Are diagnostic of collagen vascular disease
B- Contain swollen glial cells
C- Never disappear once they are formed
D- Are transudates from the superficial capillary plexus
E- Represent coagulative necrosis of the nerve fiber layer

29- The most commonly used fixative in ophthalmic pathology is
A- Gluteraldehyde
B- Formalin
C- B5
D- RPMI

30- An appropriate use of surgical frozen sections is to
A- Provide a rapid diagnosis for anxious family members
B- Freeze all of the excised tissue
C- Determine if representative tissue has been sampled
D- Determine the margins of resection of PAM with atypia

31- A giant cell with an annulus of nuclei surrounded by a lipid-filled zone is classified as a
A- Langhans giant cell
B- Foreign body giant cell
C- Tumor giant cell
D- Touton giant cell

26

32- Vitreous amyloidosis is associated with
A- Peripheral neuropathy
B- Cranial nerve paralysis
C- Lattice dystrophy
D- Peripheral vascular disease

33- Which of the following statements about Descemet’s membrane is FALSE?
A- It is developmentally derived from the neural crest.
B- It is elaborated by the corneal endothelial cells.
C- It remains static throughout life.
D- It is composed of type IV collagen.

34- Band keratopathy is characterized by
A- Randomly distributed deposits in the cornea
B- Calcium deposition within the deep corneal stroma
C- Occurrence only in patients with hypercalcaemia
D- Involvement of the epithelial basement membrane and Bowman’s layer
E- Association with other congenital malformations

35- All of the following may be associated with congenital cataracts EXCEPT
A- Anterior lenticonus
B- Cerulean cataracts
C- Aniridia
D- Rubella
E- Siderosis

36- Which of the following sites for ocular adnexal lymphoid neoplasms has the strongest
association with systemic lymphoma?
A- Eyelid
B- Palpebral conjunctiva
C- Bulbar conjunctiva
D- Orbit

37- The conjunctival melanocytic proliferation most commonly associated with subsequent
development of conjunctival malignant melanoma is
A- Ephelis
B- Nevus
C- Ocular melanocytosis
D- Primary acquired melanosis

38- Exfoliation syndrome (pseudoexfoliation) does NOT include
A- Systemic deposition of fibrillin
B- A history of infrared exposure
C- Transillumination defects of the iris
D- Deposition of a flaky material on lens zonule fibers

39- Which of the following choices best matches the description of “a lymphoid infiltrate with a
vaguely follicular pattern that includes the proliferation of monocytoid B lymphocytes and a
heterogeneous mix of small lymphocytes and plasma cells”?
A- Reactive lymphoid hyperplasia
B- Mucosa-associated lymphoid tissue (MALT)
C- Small B cell lymphoma
D- Orbital inflammatory syndrome

27

40- Which of the following statements about uveal melanoma is true?
A- Uveal melanomas rarely metastasize.
B- Melanomas of the ciliary body have the best prognosis.
C- Metastases always occur within 21/2 years of treatment.
D- Survival is directly related to tumor volume.

41- Which of the following statements about persistent hyperplastic primary vitreous is TRUE?
A- Visual prognosis is excellent.
B- Early angle-closure glaucoma is common.
C- Retinal detachment is rare.
D- The eye is usually normal in size.
E- Cataract is uncommon.

42- Medulloepitheliomas
A- Are tumors arising from surface ectoderm-derived cells
B- Are congenital lesions often containing cartilage
C- Are usually malignant in nature
D- Can be caused by trauma to the ciliary body
E- Usually metastasize to the liver

43- Which of the following tumors develops in the absence of a normal tumor-suppressor gene?
A- Choroidal melanoma
B- Melanocytoma
C- Medulloepithelioma
D- Retinoblastoma

44- The average volume of the adult vitreous cavity is
A- 3 cc
B- 4 cc
C- 5 cc
D- 6 cc

45- Which of the following statements about corneal dystrophies is TRUE?
A- They may occur unilaterally.
B- They are inherited disorders.
C- Their development often follows surgical or accidental trauma to the eye.
D- They rarely cause visual symptoms or impairment.
E- They are never associated with stromal thinning.

28
V - NEURO-OPHTHALMOLOGY (40 q)

1- The optic nerve is especially vulnerable to injury with surgery on which one of the following
sinuses?
A- Frontal
B- Ethmoid
C- Maxillary
D- Sphenoid

2- A 44-year-old female presents with gradual onset of diplopia. She has 4 mm of ptosis,
complete absence of upgaze and adduction, and a 50% reduction of abduction. The pupil is 5
mm and poorly reactive, but there is no afferent pupillary defect. Visual acuity is normal, as
are the results of external and fundus evaluations. The lesion most likely localizes to which
one of the following areas?
A- Orbit
B- Cavernous sinus
C- Brain stem
D- Neuromuscular junction

3- For the patient mentioned above, the most appropriate first procedure should be which one of
the following?
A- MRI with contrast
B- CT without contrast
C- Ultrasonography
D- Tensilon test

4- Sympathetic fibers are intimately involved with all EXCEPT which one of the following as it
courses toward the eye?
A- Abducens nerve (CN VI)
B- Trigeminal nerve (CN V)
C- Oculomotor nerve (CN III)
D- Carotid artery

5- MRI is generally superior to CT for imaging all EXCEPT which one of the following?
A- Demyelinating disease
B- Acute infarct
C- Acute hemorrhage
D- Congenital parenchymal abnormalities

6- The clinical history and findings most typical of optic nerve sheath meningiomas include all
EXCEPT which one of the following?
A- Female
B- Marked proptosis
C- Slowly progressive visual loss
D- Atrophy or swelling of the optic disc

7- Which of the following findings is NOT helpful in distinguishing optic disc drusen from true
papilledema?
A- Disc elevation
B- Capillary dilation
C- Normal peripapillary nerve fiber layer
D- Early hyperfluorescence on fluorescein angiography

29

8- An obese 23-year-old woman presents with headaches and transient visual obscurations.
Bilateral papilledema is documented. The next step should be which one of the following?
A- Start oral acetazolamide
B- Perform lumbar puncture
C- Suggest weight reduction
D- Obtain CT or MRI of the brain

9- Which of the following is the most common optic neuropathy in patients over age 50?
A- Nonarteritic anterior ischemic optic neuropathy (NAION)
B- Compressive
C- Demyelinating
D- Traumatic

10- A normal optic disc appearance is consistent with all EXCEPT which one of the following
diagnoses?
A- Leber hereditary optic neuropathy
B- Indirect traumatic optic neuropathy
C- Optic neuritis
D- Anterior ischemic optic neuropathy (AION)

11- A 24-year-old smoker presents with painless, semi-acute, progressive visual loss in his right
eye. Both eyes are normal on fundus evaluation. MRI results are normal. The affected eye
does not improve. Three months later, the vision in the patient’s left eye deteriorates. Repeat
MRI and a lumbar puncture yield negative results. The complete blood count and assays for
antinuclear antibody, fluorescent treponemal antibody absorption, angiotensin-converting
enzyme, Lyme titer, vitamin B12, and folate are normal. What is the most likely diagnosis?
A- Multiple sclerosis (MS)
B- Tobacco-alcohol amblyopia
C- Posterior ischemic optic neuropathy (PION)
D- Leber hereditary optic neuropathy (LHON)

12- Gaze-evoked amaurosis is most suggestive of which one of the following?
A- Carotid artery disease
B- Papilledema
C- Optic disc drusen
D- Orbital tumor

13- A 1-year-old child presents with monocular vertical nystagmus. What is the best course of
action?
A- Follow the case to see whether head nodding develops
B- Follow the case to see whether abnormal head position develops
C- Undertake drug toxicology screening
D- Undertake neuroimaging (preferably MRI)

14- Down-beat nystagmus is associated with all EXCEPT which one of the following?
A- Lithium
B- Tumors of the third ventricle
C- Arnold-Chiari malformation
D- Cerebellar atrophy

30

15- Convergence-retraction nystagmus is best demonstrated by:
A- Having patients rapidly converge their gaze by fixating on their nose
B- Having patients direct their gaze laterally, then re-fixate in primary position
C- Rotating an OKN drum downward
D- Having the patient look rapidly (saccade) upward

16- All EXCEPT which one of the following statements are TRUE regarding congenital ocular
motor apraxia?
A- Voluntary saccades are slow.
B- Voluntary saccades are hypometric.
C- The location of the underlying lesion is unknown.
D- Eye movements lag behind the turning head.

17- Which one of the following is indicated after aberrant regeneration in a healing oculomotor
(CN III) nerve palsy that had been presumed to be diabetic?
A- No further work-up is indicated
B- Westergren erythrocyte sedimentation rate and C-reactive protein
C- MRI, possibly MRA
D- Lumbar puncture

18- A 53-year-old female gives a 20-year history of flashing lights in only her left visual field; the
lights last 20–30 minutes and occur several times a year. There are no other neurologic
symptoms. Which one of the following is indicated?
A- MRI of the brain
B- Observation
C- Carotid ultrasonography
D- Westergren erythrocyte sedimentation rate and C-reactive protein

19- The Optic Neuritis Treatment Trial (ONTT) showed which one of the following?
A- Prompt, high-dose (1 g/day) IV corticosteroid therapy is indicated to preserve vision.
B- Oral prednisone (1 mg/kg) is sufficient treatment because high-dose IV therapy
provides no advantage in terms of visual outcome.
C- Visual prognosis remains poor, regardless of intervention.
D- IV corticosteroids may be beneficial when MRI results are consistent with multiple
sclerosis.

20- Which one of the following is NOT a cause or risk factor for pseudotumor cerebri?
A- Tetracyclines
B- Oral contraceptives
C- Weight loss
D- High consumption of dietary liver

21- Which one of the following is NOT a defined risk factor for nonarteritic anterior ischemic
optic neuropathy (NAION)?
A- Hypertension
B- Carotid occlusive disease
C- Diabetes mellitus
D- Small cup-to-disc ratio

31
22- Which one of the following statements is most TRUE about a relative afferent pupillary
defect (RAPD)?
A- An RAPD is never seen except in the setting of optic neuropathy.
B- A patient with optic neuropathy always has an RAPD.
C- An RAPD is never seen in amblyopia or retinal disease.
D- A chiasmal lesion may be associated with an RAPD.

23- Which one of the following is NOT a typical feature of the dorsal midbrain syndrome (also
known as Parinaud’s syndrome or pretectal syndrome)?
A- Lid retraction
B- Convergence-retraction nystagmus
C- Small pupils that react poorly to light and briskly to near
D- Saccadic paresis of upgaze

24- Which one of the following is TRUE about optic neuritis?
A- The majority of patients with multiple sclerosis (MS) will have optic neuritis at some
point.
B- A decrement in visual function with exposure to cold is common after optic neuritis.
C- There is no risk of MS in the patient presenting with a first isolated bout of optic
neuritis if MRI shows no lesions typical of MS.
D- Pain or discomfort on ocular rotation occurs in the minority of cases of acute optic
neuritis.

25- Which one of the following statements is FALSE?
A- Acute internuclear ophthalmoplegia in a patient older than 55 years is suspicious for
vascular disease.
B- Acute internuclear ophthalmoplegia occurring in patients 15–40 years old is
suspicious for demyelinating disease.
C- The acute onset of internuclear ophthalmoplegia and a skew deviation in a 65-year-
old patient is most likely a paraneoplastic disorder.
D- The limited adduction of internuclear ophthalmoplegia is always accompanied by
obvious abducting nystagmus.

26- Which of the following is NOT TRUE about myasthenia gravis (MG)?
A- The see-saw sign (manual elevation of the ptotic lid accompanied by ptosis of the
contralateral lid) is a manifestation of Sherrington’s law.
B- The sleep test and ice pack test are useful adjuncts in the diagnosis of MG.
C- A negative acetylcholine receptor antibody assay is not uncommon in ocular
myasthenia.
D- The minority of patients with ocular MG have thymoma.

27- Which one of the following is NOT a feature of myotonic dystrophy?
A- Hatchet facies
B- Inability to relax one’s grasp
C- Subluxed lenses
D- Ophthalmoplegia

28- Which one of the following is NOT helpful in the evaluation or treatment of Kearns-Sayre
syndrome?
A- Electrocardiogram (EKG) with a long rhythm strip
B- Mitochondrial genetic testing
C- A negative family history
D- Finding pigmentary retinopathy

32
29- Which one of the following is a cause of bilateral facial nerve palsies?
A- Miller Fisher syndrome
B- Sarcoidosis
C- Lyme disease
D- Meige syndrome

30- An otherwise healthy 18-year-old patient presents with blurred vision of the right eye. She
has uncorrected distance acuity of 20/25 OD and 20/15 OS and near acuity of 14 point OD
and 3 point OS. The examination is otherwise normal except for the pupillary findings. The
pupils are 9 mm OD and 7 mm OS in dim light, and 8.5 mm OD and 2 mm OS in bright light
and at near. The poor vision in the right eye is due to which one of the following?
A- Unexpected loss of accommodation in the right eye
B- Keratoconus
C- Optic neuritis
D- Myopic shift due to the enlarged pupil

31- Concerning the patient discussed above, if you wish to prove your working diagnosis, which
one of the following eye drops would be your best choice as a first diagnostic test?
A- Pilocarpine 1%
B- Cocaine 5%
C- Hydroxyamphetamine 1%
D- Pilocarpine 0.1%

32- A 55-year-old diabetic patient presents with double vision. Which one of the following
findings would lead you to order MRI of the brain as the initial diagnostic test?
A- Isolated pupil-sparing complete third cranial nerve palsy of 3 days’ duration
B- Right fourth cranial nerve palsy with bilateral ptosis that clearly gets worse as the day
progresses, present for 1 week
C- Isolated pupil-sparing complete third cranial nerve palsy of 8 months’ duration
D- Lid retraction OU, restriction of upgaze OU, injection over the medial and lateral recti
muscles OU, and elevation of intraocular pressure from 16 to 28 mm Hg in both eyes
with upgaze

33- A 27-year-old female with no history of medical, ocular, or neurologic problems presents
with pain on ocular rotation of the left eye, a mild visual loss for 4 days, decreased visual
acuity of 20/25 OS, and a mild sense of colors being washed out in the left eye. Aside from a
mild relative central scotoma in the left eye and a left relative afferent pupillary defect (0.6 log
units), examination results are normal. (All OD results are normal.) She is not very bothered
by her symptoms. Which of the following is the most appropriate strategy?
A- Administer 1 g of IV methylprednisolone (Solu-Medrol) daily for 3 days, then
administer 80 mg oral prednisone daily for 11 days, then see the patient in 11 days.
B- Schedule her for MRI of the brain in 1–4 days and have her return to the office the day
after the scan for reevaluation with the MRI results.
C- Send her for a neurologic consultation in 2 weeks and see her in 4 weeks.
D- Order MRI of the orbit for the next day and have her return in 48 hours.

34- A lesion in the left temporal lobe would be most likely to affect which visual field quadrant in
the left eye?
A- Upper temporal
B- Lower temporal
C- Upper nasal
D- Lower nasal

33
35- Ninety percent of isolated homonymous hemianopias are caused by which one of the
following?
A- Head trauma
B- Stroke
C- Intracranial neoplasms
D- Demyelinating disease

36- Which one of the following symptoms would be more likely with a temporal lobe lesion than
with a parietal lobe lesion?
A- Formed visual hallucinations
B- Agnosia
C- Apraxia
D- Acalculia

37- Which one of the following statements about carotid endarterectomy is NOT correct?
A- It reduces the risk of stroke in symptomatic patients with 70% or greater stenosis.
B- Stroke reduction and surgical outcomes are not related to the surgical experience of
the team at the treating medical center.
C- Endarterectomy was not beneficial for patients with moderate (50%–69%) stenosis.
D- Aspirin may be beneficial in patients with mild carotid occlusive disease.

38- A patient presents with headache, blurred vision, and a fixed mid-dilated pupil. The opposite
pupil is normal. The dilated pupil does not respond to one drop of pilocarpine 1%. What is the
most likely explanation?
A- Intracranial aneurysm
B- Vasculopathic oculomotor nerve (CN III) palsy
C- Migraine
D- Pharmacologic mydriasis

39- After a motor vehicle accident, a young man presents with unilateral marked proptosis,
diplopia, and prominent episcleral vessels. He complains of a rushing sound in his head.
Which one of the following is the most likely possibility?
A- Posteriorly draining dural shunt
B- Carotid-cavernous fistula
C- Retro-orbital hemorrhage
D- Orbital floor fracture

40- A patient presents with acute-onset right-sided headache, Horner syndrome, ipsilateral loss
of vision, diplopia, and contralateral hemiparesis. What is the best explanation?
A- Right middle cerebral artery hemorrhagic stroke
B- Basilar artery stroke
C- Spontaneous dissection of carotid artery
D- Ruptured intracranial aneurysm

34
VI - PEDIATRIC OPHTHALMOLOGY AND STRABISMUS (50 q)

1- A 7-year-old girl is brought in for evaluation of a left "lazy eye". The parents describe
intermittent outward drifting of the eye since the girl was 4 years old. Recently, the frequency
and duration of the deviation have increased. Visual acuity is 20/50 OU, which corrects to
20/20 OU with ­1.5 sphere OU. The exotropia measures 30 X' and 35 X(T). Stereopsis is 40
arc seconds. The best initial treatment of this patient is
A- single-vision glasses of ­1.5 sphere OU
B- single-vision glasses of ­3.5 sphere OU
C- alternate patching, 4 hours per day
D- patching of the right eye only, 4 hours per day

2- A 4-year-old girl is brought in for evaluation of new-onset right esotropia. There was no
antecedent viral illness or head injury. Her pediatrician has already performed CT, the results
of which were normal. The parents show you several photographs of how the child looked
before with straight eyes. In each photograph, she has a consistent face turn 15° left. The
right eye abducts fully. The most likely diagnosis is
A- congenital palsy, right cranial nerve VI
B- acute idiopathic palsy, right cranial nerve VI, resolved
C- Duane syndrome type 1 OS
D- Duane syndrome type 2 OD

3- A 5-month-old healthy full-term infant is brought in for evaluation of esotropia. The deviation
measures 50 ET' by Krimsky testing and is constant in the office. The child cross-fixates
equally well with each eye. The cycloplegic refraction is +1.0 sphere OU. Surgery should be
performed
A- immediately
B- around 1 year of age, in order to allow time for spontaneous resolution
C- between 1 and 2 years of age, so that if inferior oblique overaction or dissociated
vertical deviations develop they can be treated at the same time (ie, during one
surgery)
D- as soon as the child is old enough to permit a reliable alternate-cover test

4- A 9-month-old healthy full-term infant is brought in for evaluation of esotropia. The crossing
started variably around 6-7 months of age and now is nearly constant. Vision is central,
steady, and maintained in each eye. The deviation measures 30 ET' by alternate cover
testing. The ocular rotations show ­1 limitation of voluntary abduction OU. The cycloplegic
refraction is +4.0 sphere OU. The first step in treatment is
A- neuroimaging
B- full-time alternate occlusion
C- glasses containing +4.0 sphere OU
D- glasses containing +2.0 sphere OU

5- A 2-year-old girl is brought in for examination because her mother and brother have
neurofibromatosis type 1 and the family wishes to determine whether this child is also
affected. Her ocular examination results are normal. Which of the following statements is
TRUE?
A- The absence of Brushfield spots means that the child is unaffected.
B- Optic nerve gliomas in neurofibromatosis usually develop in infancy, so this child is
unlikely to be affected by this complication.
C- The absence of Lisch nodules does not mean that the child is unaffected.
D- The child should undergo examination under sedation to check intraocular pressure.

35
6- Vision loss in Riley-Day syndrome is most often due to
A- cataracts
B- optic nerve hypoplasia
C- amblyopia
D- corneal scarring

7- A 5-year-old child presents with a 5-day complaint of progressive right upper- and lower-lid
redness and swelling. Currently, the lids are swollen shut, tender, and warm. He was placed
on ampicillin and Benadryl by his pediatrician 3 days earlier, without noticeable improvement.
His past medical history is significant for lots of allergies and congestion. The examination is
limited because of poor cooperation, and a lid speculum is required to open the right eyelids.
There is moderate conjunctival injection; the cornea is clear and iris details are visible; the
pupil is round and reactive; the optic disc is normal. By observation, the ocular rotations are
grossly full. There is no area of broken skin on the eyelids. The left eye is normal. The next
step in treatment is to
A- ask the pediatrician to change to a stronger oral antibiotic such as
amoxicillin/clavulanate potassium (Augmentin)
B- order MRI
C- order CT
D- order a dermatologic consultation

8- All of the following conditions have a characteristic anterior-segment finding EXCEPT
A- Sickle cell disease
B- Marfan syndrome
C- Galactosemia
D- Wilson disease

9- The parents of a 7-month-old child complain of intermittent tearing OD only, beginning 3
months ago. Their pediatrician prescribed lacrimal sac massage but noticed a decreased red
reflex OD on a follow-up visit. The most likely diagnosis is
A- congenital glaucoma
B- infantile cataract
C- chlamydial conjunctivitis with corneal scarring
D- retinoblastoma

10- A 5-month-old infant has a capillary hemangioma of the right upper lid involving the
temporal third of the lid. The lid margin is deformed in an S-shape and covers about one third
of the superior pupil. The child appears amblyopic by fixation preference testing. The most
likely cause is
A- intermittent occlusion during periods of fatigue
B- intermittent occlusion when the child looks to the right
C- anisometropic amblyopia due to unequal hyperopia
D- anisometropic amblyopia due to unequal astigmatism

11- A 2-year-old child with a history of prematurity and cerebral palsy is brought in for evaluation
of possible ocular torticollis. The child maintains a frequent head posture of tilting to the left.
He can sit unaided but is not crawling or walking. On examination, his vision is central,
steady, maintained OD and central, steady, not maintained OS. There is a constant 45∆ left
esotropia, and bilateral inferior oblique muscle overaction is greater OS than OD. Which of
the following best explains the torticollis?
A- It is non-ocular.
B- It is due to congenital bilateral superior oblique palsies.
C- It is an effort to avoid the left inferior oblique overaction.
D- It is due to a V pattern.

36
12- Dissociated vertical deviation (DVD) is characterized by which of the following features?
A- extorsion or horizontal component as the eye moves vertically
B- upward movement to fixate of the adducting eye in lateral gaze
C- essentially identical movements of the eyes in the alternate-cover test in all positions
of gaze
D- (a) and (b)
E- (a) and (c)

13- Tucking the superior oblique tendon
A- is appropriate to correct superior oblique muscle palsy
B- can result in Brown syndrome
C- is the procedure of choice when the symptoms and measurements indicate principally
a torsional misalignment
D- (a) and (b)
E- (b) and (c)

14- Which feature(s) below are not typically found in Brown syndrome?
A- hyperdeviation in the lower field of gaze
B- exotropia in straight upgaze
C- incomplete elevation in straight upgaze
D- esotropia in downgaze
E- (a) and (d)

15- Which of the following statements about strabismus secondary to thyroid ophthalmopathy is
FALSE?
A- It can be restrictive.
B- It can be caused by extraocular muscle weakness.
C- It usually is surgically corrected early after onset.
D- It is unrelated to the degree of thyroid function.
E- (b) and (c) are correct.

16- Congenital dacryocele
A- presents with a mass above the medial canthal ligament
B- usually responds to systemic antibiotics alone
C- can be associated with an intranasal mucocele
D- is best treated with incision and drainage through the skin
E- usually indicates stenosis of the bony nasolacrimal canal

17- The electroretinogram (ERG) usually appears normal in the following conditions EXCEPT
A- optic nerve hypoplasia
B- cortical visual impairment
C- achromatopsia
D- delayed visual maturation
E- (a) and (c)

18- Which of the following statements is FALSE?
A- Epiblepharon is well-tolerated and only occasionally requires surgical correction.
B- Telecanthus indicates increased separation between the bony orbits.
C- Amblyopia resulting from ptosis is usually a result of induced astigmatism rather than
occlusion.
D- The blepharophimosis syndrome is often inherited in an autosomal dominant fashion.

37
19- Which of the following choroidal lesions may be associated with adenocarcinoma of the
colon?
A- choroidal osteoma
B- combined hamartoma of the retina and retinal pigment epithelium
C- melanocytoma
D- congenital hypertrophy of the retinal pigment epithelium

20- Which of the following diagnostic tests should be performed on all patients with
retinoblastoma?
A- CT of the head and orbits
B- total-body CT
C- aspiration of intraocular fluid
D- bone marrow biopsy

21- Regarding the treatment of retinoblastoma, which of the following statements is true?
A- Enucleation is no longer an appropriate treatment for retinoblastoma.
B- External beam irradiation results in a much higher incidence of secondary tumor
formation.
C- Cryotherapy and laser therapy can be used for tumors that are 5 mm in thickness.
D- Plaque radiotherapy should not be used for retinoblastomas that fail to respond to
other methods of treatment.

22- Which of the following statements is FALSE?
A- Juvenile rheumatoid arthritis is the most commonly identified cause of childhood
uveitis.
B- Steroid administration in children is less likely to result in cataracts or glaucoma than
in adults.
C- Toxoplasmosis is the most commonly identified infectious cause of childhood uveitis.
D- Toxocariasis is usually caused by Toxocara canis.

23- Which of the following is NOT a complication of steroid administration in children?
A- angle-closure glaucoma
B- bone fractures
C- death
D- altered mental status

24- Which of the following statements regarding the pathophysiology of amblyopia is TRUE?
A- Changes in the nerve fiber layer of the retina are characteristic of strabismic
amblyopia.
B- The cells in the medial geniculate body corresponding to the amblyopic eye may be
smaller and less intensely staining than those corresponding to the sound eye.
C- The visual acuity of an amblyopic eye may be better when measured in the presence
of contour interaction than if measured with isolated optotypes.
D- The most significant change in the visual cortex of an amblyopic patient is loss of
binocular cells, cells that are responsive to stimulation from either eye.
E- The sensitive period for the development of deprivation amblyopia begins earlier and
lasts longer than that for strabismic or anisometropic amblyopia.

38
25- Which one of the following statements regarding accommodative esotropia is FALSE?
A- In the gradient method of determining the AC/A ratio, plus or minus lenses can be
used to evaluate the effect of accommodation on the deviation at a given distance.
B- In the heterophoria method for determining the AC/A ratio, measurements of the
deviation at distance and near are compared.
C- It is generally acceptable to perform surgery for fully accommodative esotropia as long
as the patient has good fusion.
D- Executive or flat-top bifocals are preferred over progressive bifocals in young children
with nonrefractive accommodative esotropia.
E- Miotic agents can be used diagnostically in accommodative esotropia or as a short-
term substitute for glasses.

26- Regarding patients with congenital nasolacrimal duct obstruction, which of the following
statements is FALSE?
A- Obstruction of the nasolacrimal duct occurs in 2%-4% of full-term infants during the
first 2 weeks of life.
B- Topical antibiotics, lacrimal massage, and nasal decongestants are therapies in the
initial treatment of these children.
C- In-office probing, without general anesthesia, can be done anytime during the first
year of life.
D- The success rate of probe and irrigation alone drops to 33% if not done by 2 years of
age.

27- Which of the following conditions is NOT characteristically associated with childhood
glaucoma?
A- Sturge-Weber syndrome
B- aniridia
C- galactosemia
D- Lowe syndrome

28- A 2-month-old boy presents with epiphora, photophobia, and a band of cloudiness of the
right cornea that does not obscure the pupil. Appropriate evaluation should include
A- corneal biopsy with Oil Red-O-stain
B- examination under anesthesia with IOP measurement at the end of the case
C- refraction
D- conjunctival scraping for chlamydia

29- You examine a 1-year-old boy with bilateral cataracts. Appropriate evaluation includes
A- chromosomal analysis and examination of female relatives
B- serum copper level
C- urine protein level
D- red blood cell galactose-1-phosphate uridyl transferase level

30- Which of the following is LEAST likely to cause a subluxation of the lens?
A- Marfan syndrome
B- homocystinuria
C- hyperlysinemia
D- Weill-Marchesani syndrome

39
31- A 1-year-old child has a 3-month history of intermittent, rapid, asymmetrical, fine nystagmus.
Ophthalmologic and neurologic examination results are otherwise normal. Further evaluation
should include MRI of the
A- cerebellum
B- chiasmal area
C- brain stem
D- foramen magnum

32- Which of the following is NOT a major criterion for the diagnosis of neurofibromatosis type
1?
A- sphenoid wing dysplasia
B- posterior subcapsular cataract
C- single plexiform neurofibroma
D- optic nerve glioma

33- Which of the following phakomatoses is NOT inherited?
A- incontinentia pigmenti (Bloch-Sulzberger syndrome)
B- ataxia-telangiectasia (Louis-Bar syndrome)
C- racemose angiomatosis (Wyburn-Mason syndrome)
D- tuberous sclerosis (Bourneville disease)

34- Which of the following is TRUE concerning retinal angiomas associated with von Hippel-
Lindau disease?
A- Their peak clinical incidence coincides with the peak incidence of cerebellar
hemangioblastomas.
B- The tumors are rarely bilateral.
C- Vision loss is usually secondary to lipid accumulation and serous retinal detachment.
D- Cryotherapy, laser photocoagulation, or both are ineffective in the treatment of these
lesions.

35- The most common horizontal deviation associated with craniosynostosis syndromes is
A- A-pattern esotropia
B- A-pattern exotropia
C- V-pattern esotropia
D- V-pattern exotropia

36- Crouzon syndrome is most often associated with
A- plagiocephaly
B- brachycephaly
C- scaphocephaly
D- syndactyly

37- Findings associated with Goldenhar syndrome include all of the following EXCEPT
A- cleft palate
B- epibulbar dermoid
C- eyelid coloboma
D- Duane syndrome

38- Which antibiotic should be given for suspected neonatal Neisseria conjunctivitis?
A- intravenous ceftriaxone
B- intravenous penicillin
C- oral amoxicillin
D- topical gentamicin
E- topical ciprofloxacin

40
39- Which of the following is NOT a major cause of Parinaud’s oculoglandular syndrome?
A- Bartonella henselae
B- Borrelia burgdorferi
C- Francisella tularensis
D- Mycobacterium tuberculosis

40- Which is a feature of orbital, NOT preseptal, cellulitis?
A- eyelid edema
B- tenderness to palpation
C- proptosis
D- conjunctivitis

41- Which of the following is NOT a characteristic of Sturge-Weber syndrome?
A- glaucoma
B- choroidal hemangioma
C- cerebral calcification
D- posterior subcapsular cataract

42- Overcorrections following strabismus surgery
A- can be avoided with adjustable sutures
B- can occur when the resected muscle slips or is lost after surgery
C- are less common than undercorrections
D- frequently cause diplopia in young children

43- Scleral perforations during strabismus surgery in children
A- are frequently followed by retinal detachment
B- are frequently treated with laser therapy or cryopexy
C- occur in about 10% of strabismus surgeries
D- frequently cause postoperative endophthalmitis

44- Localized conjunctival injection and chemosis noted several days postoperatively at the site
of eye muscle surgery may be caused by
A- suture allergy
B- poor closure of the conjunctival wound
C- conjunctival inclusion cyst
D- all of the above

45- Anterior segment ischemia following eye muscle surgery
A- is especially common following muscle transposition procedures
B- can occur following surgery on two rectus muscles
C- typically presents with iritis and an irregular pupil
D- all of the above

46- Botulinum toxin is particularly useful for
A- large-angle exotropia
B- mechanical restrictions
C- small residual angles following strabismus surgery
D- dissociated vertical deviations

41
47- A 3-year-old boy was brought to the emergency room following an injury to the right eye.
The history states that he wandered too close when his father was practicing his golf swing.
The lids are ecchymotic and cannot be opened. The child is inconsolably terrified. You should
A- call Child Protective Service
B- get a papoose board to restrain the child and open the lids with a speculum to
investigate for ocular injury
C- prepare the child for an examination under anesthesia
D- send the child home for reexamination in several days, when the swelling has
subsided

48- All of the following are characteristic of the shaken baby syndrome EXCEPT
A- the child is younger than 12 months
B- parenchymal brain damage and intracranial hemorrhage are common
C- ocular adnexae and anterior segments are typically involved
D- retinal hemorrhage is typical, especially in the posterior pole

49- All of the following suggest an occult perforating injury EXCEPT
A- an eyelid laceration with intact conjunctiva
B- an irregular pupil
C- loss of ocular rotations
D- all of the above suggest perforation

50- Hyphema following blunt trauma in a child
A- should be treated on an inpatient basis, with bilateral patching and bed rest
B- should be treated with long-acting cycloplegics and topical steroids on an outpatient
basis
C- is more common in children with sickle cell hemoglobin
D- should be evacuated if a total hyphema in a young child persists more than 4 or 5
days

42
VII - ORBIT, EYELIDS AND LACRIMAL SYSTEM (45 q)

1- This nerve to an extraocular muscle does NOT pass through the muscle cone on entering the
orbit:
A- CN III
B- CN IV
C- CN V
D- CN VI
E- CN VII

2- Which is TRUE regarding orbital anatomy?
A- The lacrimal gland fossa is located within the lateral orbital wall.
B- The optic canal is located within the greater wing of the sphenoid bone.
C- The medial wall of the optic canal is formed by the lateral wall of the sphenoid sinus.
D- The nerve to the inferior rectus muscle travels anteriorly along the medial aspect of
the muscle and innervates the muscle on its posterior surface.

3- The five major branches of the facial nerve include the temporal, buccal, marginal
mandibular, cervical, and
A- Temporal parietal
B- Zygomatic
C- Infraorbital
D- Zygomaticofacial

4- What structure is deep to the plane of the facial nerve branches in the lower face?
A- Masseter muscle
B- Parotidomasseteric fascia
C- Deep temporal fascia
D- Parotid gland

5- The superior transverse ligament is also referred to as
A- Lockwood’s ligament
B- Sommerring’s ligament
C- The ROOF
D- Whitnall’s ligament

6- Compared to CT scanning, MRI scanning provides better
A- View of bone and calcium
B- View of the orbital apex and orbitocranial junction
C- Elimination of motion artifact
D- Comfort for claustrophobic patients
E- Safety to patients with prosthetic implants

7- In patients with a facial nerve paralysis, all of the following characteristics may be present
except:
A- Eyebrow ptosis
B- Blepharoptosis
C- Lower eyelid ectropion
D- Epiphora
E- Ocular exposure symptoms

43
8- A 1-year-old presents with a round, well-demarcated mass at the superotemporal rim. The
lesion has been present since birth. The most likely diagnosis is
A- Rhabdomyosarcoma
B- Neurofibroma
C- Dermoid cyst
D- Capillary hemangioma
E- Metastatic Ewing sarcoma

9- A 65-year-old woman presents with a progressively enlarging mass in the right inferior orbit.
Distraction of the lower eyelid reveals a “salmon patch” appearance to the fornix. The most
likely diagnosis is
A- Reactive lymphoid hyperplasia
B- Lymphoma
C- Sebaceous carcinoma
D- Melanoma
E- Apocrine hidrocystoma

10- Which of the following signs is most likely to be present in a patient with Graves’s
ophthalmopathy?
A- Exophthalmos
B- External ophthalmoplegia
C- Eyelid retraction
D- Optic neuropathy

11- Subperiosteal abscess of the orbit in adults is more likely than in children to
A- Drain spontaneously
B- Respond to single antibiotic therapy
C- Be polymicrobial
D- Arise from the ethmoid sinus
E- Arise from acute sinusitis

12- In exploring upper eyelid trauma with a full-thickness laceration involving the eyelid margin,
the physician must be aware of the order in which the anatomical structures are normally
encountered. The correct order is
A- Skin, preaponeurotic fat, septum, orbicularis muscle, levator aponeurosis, Müller’s
muscle, conjunctiva
B- Skin, orbicularis muscle, preaponeurotic fat, Müller’s muscle, levator aponeurosis,
conjunctiva
C- Skin, preaponeurotic fat, orbicularis muscle, septum, levator aponeurosis, Müller’s
muscle, conjunctiva
D- Skin, orbicularis muscle, septum, preaponeurotic fat, levator aponeurosis, Müller’s
muscle, conjunctiva

13- Which of the following statements about the relationship of Graves ophthalmopathy with
thyroid status is true?
A- Ophthalmopathy resolves after hyperthyroidism is adequately treated.
B- Ophthalmopathy generally develops prior to the diagnosis of thyroid dysfunction.
C- Serum thyroid-stimulating hormone level is a good marker for the intensity of
ophthalmopathy.
D- Ophthalmopathy may develop even though a patient is euthyroid.

44
14- Congenital Horner syndrome is associated with miosis, ptosis, anhidrosis, and
A- Chemosis
B- Jaw-winking
C- Lagophthalmos
D- Iris hypopigmentation

15- The most common location for orbital lymphoma is
A- The extraocular muscles
B- Retro-orbital fat
C- Lacrimal fossa
D- Orbital apex
E- Orbital floor

16- Biopsy of an orbital mass is read as “reactive lymphoid hyperplasia.” The appropriate
treatment is
A- Systemic steroids
B- Radiation with a dose of 1500–2000 cGy
C- Systemic chemotherapy
D- Complete surgical excision
E- Cryotherapy

17- All of the following are true regarding orbital cellulitis, EXCEPT:
A- Most cases are associated with an underlying sinusitis.
B- Infections in children tend to be single organism.
C- Subperiosteal abscesses, especially in children, require drainage.
D- After initiation of appropriate antibiotic therapy a subperiosteal abscess may enlarge
before responding.

18- A 20-year-old man is struck over the right eye, and radiography shows a fracture of the right
orbital floor. Forced traction testing is equivocal because of poor cooperation. Four days after
injury, 3 mm of right exophthalmos is present, and movements of the eye are restricted in
upgaze, downgaze, and horizontal gaze. Treatment at that time should be
A- Conjunctival incision through the inferior fornix, with examination of the fracture
B- Caldwell-Luc incision and packing of the maxillary sinus
C- Skin incision over the inferior orbital rim and covering of the fracture defect with a
plastic plate
D- Skin incision beneath the eyelashes and covering of the fracture defect with a plastic
plate
E- None of the above

19- Which of the following features is most likely to be found on an orbital CT scan of a patient
with Graves’s ophthalmopathy?
A- An increased amount of orbital fat in the presence of normal-sized extraocular
muscles
B- Diffuse, fusiform enlargement of the extraocular muscle belly and tendon
C- Pressure erosion of the lateral orbital rim from enlarged muscles
D- Chronic ethmoid and maxillary sinusitis

20- The most important determinant in selecting a corrective procedure for any type of ptosis is
A- Vertical height of the palpebral fissure
B- Age of the patient
C- Amount of levator function
D- Duration of the ptosis
E- Position of the upper eyelid margin relative to the corneal limbus

45
21- A 70-year-old woman has 4 mm of right upper eyelid ptosis and 1 mm of left upper eyelid
retraction. She has a high eyelid crease in the right upper eyelid with normal levator function
of both upper eyelids. The treatment of choice is
A- A moderated internal tarsoconjunctival resection (Fasanella-Servat operation) on the
right upper eyelid
B- A moderate levator recession of the left upper eyelid
C- A levator aponeurosis advancement on the right upper eyelid
D- A posterior-approach, standard müllerectomy on the right upper eyelid
E- A frontalis muscle suspension on the right upper eyelid using a silicone rod to allow
postoperative adjustment

22- Risk factors for developing basal cell carcinoma include all of the following EXCEPT:
A- History of smoking
B- Excessive sun exposure in the first two decades of life
C- Brown irides
D- Blond hair
E- Celtic ancestry

23- In the evaluation of a child with unilateral exophthalmos, which assumption is correct?
A- Cavernous hemangiomas are among the most common benign orbital tumors in
children.
B- Thyroid ophthalmopathy is the most common cause of unilateral exophthalmos among
children.
C- Neurofibroma is the malignant tumor that most commonly produces exophthalmos
among children.
D- Optic nerve meningiomas are more common than gliomas in children.
E- None of the above.

24- Neurofibromatosis type 1 is associated with all of the following EXCEPT:
A- Skin lesions known as adenoma sebaceum
B- Café-au-lait skin lesions
C- Plexiform neurofibromas in the eyelids
D- Optic nerve glioma
E- Cutaneous neurofibromas

25- Essential blepharospasm is usually characterized by all of the following EXCEPT:
A- Unilaterality
B- Age of onset usually over 50 years
C- Visual incapacitation
D- Obscure etiology
E- Involuntary spasms of the orbicularis muscle

26- Entropion repair of the lower eyelid may utilize any of the following techniques EXCEPT:
A- Lower eyelid retractor advancement
B- Lateral canthal tightening
C- Taping of the eyelid
D- Mucous membrane grafting to the posterior eyelid
E- Skin grafting

27- All of the following are TRUE regarding invasive aspergillus infections of the orbit EXCEPT:
A- Bone destruction is not seen.
B- Corticosteroids may produce an initial clinical improvement.
C- An adjacent sinusitis is usually present.
D- Septate branching hyphae of uniform width are seen histologically.

46
28- A 40-year-old woman presents with a progressively enlarging clear cystic mass along the
eyelid margin. The most likely diagnosis is
A- Epidermal inclusion cyst
B- Apocrine hidrocystoma
C- Syringoma
D- Trichofolliculoma
E- Basal cell carcinoma

29- A 3-year-old girl was bitten by her pet dog. A 6-mm-wide block of upper eyelid margin is
hanging by a thread of tissue. This block of tissue left a defect in the upper eyelid
approximately the same size as the tissue itself. The best treatment would be to
A- Send the block of tissue to pathology and repair the defect by approximating the two
margins
B- Send the block of tissue to pathology and repair the defect by making a lateral
canthotomy so that the skin edges can be approximated
C- Repair the eyelid by sewing the block of tissue into its normal anatomic position
D- Send the tissue to pathology and repair the defect by a transfer of tissue from the
lower eyelid
E- Keep the tissue under refrigeration for later use if necessary and close the defect by
approximation of the wound edges and lateral canthotomy

30- A 74-year-old woman presents with a 2-year history of a painless, progressively enlarging
mass in the central aspect of the upper eyelid. This has resulted in distortion of the eyelid
margin and loss of eyelashes. The most likely diagnosis is
A- Sebaceous gland carcinoma
B- Squamous cell carcinoma
C- Amelanotic melanoma
D- Basal cell carcinoma
E- Dermal nevus

31- The majority of orbital lymphomas
A- Are polyclonal proliferations
B- Are T-cell tumors
C- Are systemic at presentation
D- Are well differentiated
E- Involve both orbits

32- Which of the following statements about the levator palpebrae muscle and its aponeurosis is
FALSE?
A- Levator muscle fibers are sparse in most cases of congenital ptosis.
B- The orbital (deep) and palpebral (superficial) portions of the lacrimal gland are
separated by the aponeurosis.
C- The orbital septum fuses with the aponeurosis in the upper eyelid.
D- Whitnall’s ligament is a check ligament of the muscle.
E- None of the above

33- Six hours after a bilateral blepharoplasty, the patient complains of sudden pain near the
right eye. The dressings are removed and the right eyelids are tense and ecchymotic. The
first step would be to
A- Open the wound to release a possible retrobulbar hemorrhage
B- Consider the possibility of a cavernous sinus thrombosis and check corneal sensation
C- Measure visual acuity and check pupillary responses
D- Begin treatment with ice packs
E- Begin treatment with warm compresses

47
34- A 75-year-old woman complains of tearing and discharge. Irrigation of the lower canaliculus
produces mucopurulent reflux. All of the following are TRUE EXCEPT:
A- Jones testing will not reveal dye in the nose.
B- There is probably a common canalicular block.
C- The condition is not likely to resolve with a course of antibiotics.
D- The most likely diagnosis is a lacrimal duct obstruction.
E- The correct treatment is dacryocystorhinostomy.

35- A 14-month-old child has had tearing and discharge from the right eye since birth. Which of
the following statements is TRUE?
A- Dye disappearance testing is likely to show no asymmetry.
B- This condition is likely to resolve spontaneously.
C- The appropriate treatment is nasolacrimal duct probing.
D- Punctal abnormalities are likely to be the cause.
E- Jones I and Jones II tests are necessary to make the diagnosis.

36- Blepharophimosis is generally associated with all of the following EXCEPT:
A- Ptosis
B- Epicanthus inversus
C- Distichiasis
D- Ectropion
E- Telecanthus

37- A 30-year-old man received a knife wound involving the right upper eyelid 1 hour before
your examination. The patient is awake and alert. A 15-mm-long laceration is present 12 mm
above the lash line. The patient has 7 mm of ptosis on the right side, and there is minimal
swelling. After appropriate evaluation of the globe, the best treatment is to
A- Repair skin laceration and wait several months for ptosis to resolve
B- Keep wound clean and wait for swelling to subside in 2–3 days before attempting
repair
C- Repair muscle layer and skin layer separately
D- Explore the wound to examine the levator aponeurosis and attempt to reattach it if it is
severed from the tarsus, after which the skin and muscle should be repaired
E- Repair skin laceration and tape eyelid to brow for several days to promote healing

38- The management of rhabdomyosarcoma of the orbit usually involves
A- Lumbar puncture to rule out central nervous system metastasis
B- Exenteration of the orbit
C- Enucleation and orbital radiation
D- Systemic chemotherapy and orbital radiation
E- Radical neck dissection if cervical lymph nodes are involved

39- Which of the following statements about medial canthal trauma is correct?
A- It is always associated with a history of direct, sharp trauma to the medial canthal soft
tissues.
B- It occasionally involves the nasolacrimal duct but frequently spares the canalicular system.
C- It must be repaired within 6 hours of injury.
D- It may result in telecanthus if not repaired.
E- Delayed lacrimal drainage repair (>6 months) would probably require a standard DCR.

48
40- Indications for repair of orbital blowout fracture include all of the following EXCEPT:
A- Cosmetically unacceptable enophthalmos
B- Fractures involving more than half of the orbital floor
C- Inferior rectus weakness
D- Pain and oculo-cardiac reflex on upgaze
E- Significant inferior rectus entrapment

41- The temporal pocket dissection for standard endoscopic brow lift is performed immediately
below what layer?
A- Deep temporal fascia
B- Temporo-parietal fascia
C- Frontalis muscle
D- Galea aponeurotica

42- In a subcutaneous rhytidectomy with SMAS, the tension of the lift is placed on which layer?
A- Skin
B- Osteo-cutaneous ligaments
C- Superficial musculo-aponeurotic system
D- Periosteum

43- Cicatricial ectropion is generally associated with
A- Trichiasis
B- Anterior lamellar shortage
C- Blepharospasm
D- Symblepharon

44- All of the following are TRUE regarding optic nerve tumors EXCEPT:
A- Optic nerve gliomas in children are associated with neurofibromatosis type 1.
B- Optic nerve meningiomas in children are associated with neurofibromatosis type 2.
C- Radiation therapy is an accepted therapy for optic nerve sheath meningiomas.
D- Optic nerve gliomas of childhood can be malignant.

45- In cases of implant extrusion following enucleation or evisceration all of the following are
TRUE EXCEPT:
A- Early extrusion is associated with an implant that is too small.
B- Early extrusion is associated with poor wound closure.
C- Late implant extrusion can be associated with tumor recurrence.
D- Late implant extrusion can be associated with a conjunctival cyst.

49
VIII - EXTERNAL DISEASE AND CORNEA (43 q)

1- Which of the following associations is INCORRECT?
A- Mesectoderm—corneal endothelium
B- Mesectoderm—corneal stroma
C- Type IV collagen—Bowman’s layer
D- Type IV collagen—Descemet’s membrane

2- The patient with which of the following diagnoses would likely have the best visual acuity?
A- Primary familial amyloidosis
B- Reis-Bücklers dystrophy
C- Lattice dystrophy, type I
D- Central cloudy dystrophy of Franñois

3- A 55-year-old patient presents in your office with complaints of decreased vision. On
examination, you detect severe anterior basement membrane dystrophy and a cataract.
Which one of the following tests would be MOST helpful to you to determine whether the
main cause of decreased vision is the corneal pathology or the cataract?
A- Potential acuity meter
B- Corneal topography
C- Contact lens over-refraction
D- Glare testing

4- Follicular conjunctivitis is associated with all of the following conditions EXCEPT:
A- Herpes simplex virus conjunctivitis
B- Adult inclusion conjunctivitis
C- Drug-induced conjunctivitis
D- Allergic conjunctivitis

5- Which of the following statements regarding topical fluorescein is FALSE?
A- It is a nontoxic, water-soluble dye
B- It is used to detect disruption of intercellular junctions
C- It can be used to detect subtle epithelial irregularity even without an epithelial defect
D- It has antiviral properties

6- Which of the following statements regarding the cornea is TRUE?
A- The cornea is a spherical surface with an average central refractive power of 43 D.
B- The refractive index of the corneal stroma is 1.336.
C- The tear-cornea interface is responsible for most of the eye’s refractive power.
D- A keratoscopic image of high with-the-rule astigmatism would appear as an ellipse
with the long axis oriented horizontally.

7- Which of the following statements regarding the ocular surface is FALSE?
A- The mucin layer is the innermost layer of the tear film, is hydrophobic, and is produced
by conjunctival goblet cells.
B- Ocular rosacea results in a decreased tear breakup time.
C- Blinking is important in the release of meibomian gland lipid.
D- The tear film provides the cornea with essential nutrients and oxygen, and leukocytes
gain access to the cornea through the tear film.

50
8- All of the following suggest aqueous tear deficiency EXCEPT:
A- Decreased tear lysozyme levels
B- Decreased lactoferrin levels
C- Decreased tear film osmolarity
D- Tear meniscus between the globe and the lower eyelid of 0.3 mm
9- Which of the following is the MOST common corneal finding of sarcoidosis?
A- Calcific band keratopathy
B- Nummular keratitis
C- Thickening of Descemet’s membrane
D- Deep stromal vascularization

10- Patients who complain of increased symptoms first thing in the morning could have any of
the following conditions EXCEPT:
A- Anterior basement membrane dystrophy
B- Fuchs endothelial dystrophy
C- Floppy lid syndrome
D- Keratoconjunctivitis sicca

11- All of the following conditions could cause xerophthalmia EXCEPT:
A- Chronic alcoholism
B- Cystic fibrosis
C- Bowel resection
D- Glomerulonephritis

12- All of the following organisms can invade an intact corneal epithelium EXCEPT:
A- Neisseria meningitides
B- Corynebacterium diphtheriae
C- Shigella
D- Pseudomonas aeruginosa

13- Which of the following viruses is transmissible even after medical instrumentation is cleaned
with alcohol?
A- Herpes simplex virus
B- Adenovirus
C- Human immunodeficiency virus
D- Epstein-Barr virus

14- Which of the following statements regarding Acanthamoeba is FALSE?
A- Two potentially infectious forms of Acanthamoeba exist: the motile trophozoite and the
dormant cyst.
B- Acanthamoeba can be grown on blood agar.
C- Acanthamoeba can be visualized with calcofluor white, acridine orange, or the Giemsa
stain.
D- Acanthamoeba keratitis can appear as a pseudodendrite early in its course.

15- All of the following can be seen with ocular adenoviral infection EXCEPT:
A- Preauricular lymphadenopathy
B- Large central geographic corneal erosions
C- Multifocal subepithelial infiltrates
D- Enlarged corneal nerves

51
16- Which of the following statements regarding the Herpetic Eye Disease Study (HEDS) is
FALSE?
A- It demonstrated that topical corticosteroids given together with a prophylactic antiviral
reduce persistence or progression of stromal inflammation and shorten the duration of
herpes simplex stromal keratitis.
B- It showed that long-term suppressive oral acyclovir therapy reduces the rate of
recurrent HSV keratitis and helps to preserve vision.
C- It showed some additional benefit of oral acyclovir in treating active HSV stromal
keratitis in those patients also receiving concomitant topical corticosteroids and
trifluridine.
D- It demonstrated that oral acyclovir did not appear to prevent subsequent HSV stromal
keratitis or iritis when it was given briefly along with trifluridine during an episode of
epithelial keratitis.

17- A healthy 60-year-old man presents with a 2-day history of a painful rash on the right side of
his forehead extending down to the eyelids. A vesicular skin lesion is also seen near the tip of
his nose. Which of the following therapies would be most appropriate?
A- Topical trifluridine 1% drops 8 times per day for 14 days
B- Oral famciclovir 500 mg two times per day for 10 days
C- Oral valacyclovir 1000 mg three times per day for 10 days
D- Oral acyclovir 800 mg three times per day for 10 days

18- A 9-year-old boy with a history of atopy presents with a seasonally recurrent bilateral
conjunctivitis and complains of blurred vision for 1 week. Giant papillae are seen upon lid
eversion. All of the following could also be seen on the slit-lamp exam EXCEPT:
A- Vascular pannus and punctate epithelial erosions involving the superior cornea
B- An oval epithelial ulceration with underlying stromal opacification in the central cornea
C- Limbal follicles
D- Conjunctival symblephara

19- Which of the following statements regarding graft-versus-host disease (GVHD) is FALSE?
A- It is a relatively common complication of allogeneic bone marrow transplantation in
which the grafted cells can attack the patient’s tissues.
B- Conjunctival inflammation and severe sicca are the main features.
C- Cicatricial lagophthalmos can occur.
D- Aggressive lubrication is adequate even in severe cases of GVHD.

20- Which of the following infectious agents can be linked to interstitial keratitis?
A- Herpes simplex virus
B- Herpes zoster virus
C- Chlamydia trachomatis
D- All of the above

21- Which of the following associations regarding location of stem cells is INCORRECT?
A- Cornea—limbus
B- Conjunctiva—basal conjunctival epithelium most concentrated in the bulbar
conjunctiva
C- Epidermis—rete ridges
D- All are correct
22- Which of the following conditions has been associated with limbal stem-cell dysfunction?
A- Aniridia
B- Contact lens wear
C- Atopic keratoconjunctivitis
D- All are correct

52
23- Surgical treatment of squamous cell carcinoma should include all of the following EXCEPT:
A- Excision of conjunctiva 1 mm beyond the clinically apparent margins of the tumor
B- Resection of a thin lamellar scleral flap beneath the tumor
C- Treatment of the remaining sclera with absolute alcohol
D- Cryotherapy applied to the conjunctival margins

24- A 25-year-old Japanese woman presents with slate gray pigmentation of the episclera of the
right eye with associated pigmentation of the periocular skin. Which of the following
statements applies to her?
A- Malignant melanoma can develop in the skin, uvea, or orbit, but not in the conjunctiva.
B- Her lifetime risk of uveal melanoma is about 1 in 400, significantly greater than the risk
of the general population.
C- The risk of malignant transformation is the same in all patients regardless of their
complexion.
D- This condition is always unilateral.

25- Which of the following would signal possible malignant transformation of primary acquired
melanosis?
A- Nodularity
B- Enlargement
C- Increased vascularity
D- All of the above

26- Confirmed ocular teratogens with effects on the anterior segment other than cataract
include all of the following EXCEPT:
A- Maternal diabetes
B- Radiation
C- Maternal ingestion of anti-seizure medications and retinoic acid
D- Maternal use of topical Viroptic in the first trimester

27- Which of the following statements regarding nanophthalmos is FALSE?
A- Patients have a crowded anterior segment, which can lead to angle-closure glaucoma.
B- Peripheral choroidal effusions can occur spontaneously.
C- Large choroidal effusions occur during anterior segment surgery, possibly secondary
to impaired venous outflow through the vortex veins as a result of scleral inelasticity.
D- An extremely low power intraocular lens is required in these patients to achieve
emmetropia.

28- All of the following conditions result in corneal clouding at birth EXCEPT:
A- Congenital glaucoma
B- Congenital hereditary endothelial dystrophy with autosomal recessive inheritance
(CHED 2)
C- Congenital hereditary endothelial dystrophy with autosomal dominant inheritance
(CHED 1)
D- Birth trauma

29- Which of the following corneal dystrophies recurs MOST frequently in a corneal graft?
A- Granular
B- Macular
C- Lattice
D- Fuchs

53

30- Which of the following statements regarding Schnyder crystalline corneal dystrophy is
FALSE?
A- It is a rare, slowly progressive, autosomal dominant stromal dystrophy.
B- Central subepithelial crystals are seen in nearly 100% of patients later in life.
C- Decreased corneal sensation also occurs.
D- Dense corneal arcus lipoides is characteristic.

31- Which of the following is NOT a cause of calcific band keratopathy?
A- Hereditary transmission
B- Juvenile rheumatoid arthritis
C- Hypercalcemia caused by sarcoidosis
D- Decreased serum phosphorus with normal serum calcium

32- All of the following medications or diseases can be associated with cornea verticillata
EXCEPT:
A- Chlorpromazine
B- Chloroquine
C- Gaucher disease
D- Fabry disease

33- What is the best treatment course for a patient on amiodarone with cornea verticillata?
A- Immediate cessation of the drug
B- Concomitant administration of a steroid drop to decrease corneal pathology
C- Placement of punctal plugs
D- Observation

34- Which of the following can be related to spontaneous hyphema?
A- Herpetic disease
B- Retinoblastoma
C- Juvenile xanthogranuloma
D- All of the above

35- A 33-year-old man was involved in a motor vehicle accident that resulted in a broken
windshield. Slit-lamp examination reveals multiple glass foreign bodies embedded at various
levels of the stroma. All wounds were Seidel negative. Which of the following is the most
appropriate treatment strategy?
A- The patient should be taken to the operating room and all superficial and deep
fragments should be removed immediately.
B- All exposed glass fragments should be removed and deeper fragments left in place.
C- Ointment should be placed in the eyes since superficial and deep fragments will
eventually be extruded with time.
D- Eyes should be patched, with follow-up the next day.

36- The principal indications for a conjunctival flap include all of the following EXCEPT:
A- Neurotrophic keratopathy
B- Stromal herpes simplex viral keratitis
C- Painful bullous keratopathy in a patient who is not a good candidate for penetrating
keratoplasty
D- Small corneal perforation

54

37- All of the following are contraindications for use of donor cornea tissue EXCEPT:
A- Posterior choroidal melanoma
B- Leukemia
C- Previous PRK for correction of 2 D of myopia
D- Active septicemia

38- All of the following are appropriate therapeutic approaches to deal with a small wound leak
(with no anterior chamber shallowing) seen on the first postoperative visit after penetrating
keratoplasty EXCEPT:
A- Patching
B- Resuturing of wound
C- Use of aqueous inhibitor
D- Placement of therapeutic contact lens

39- All of the following may be signs of corneal graft rejection EXCEPT:
A- Radial perineuritis
B- Subepithelial infiltrates
C- Linear keratic precipitates
D- Elevated linear epithelial ridge

40- Currently, the most prevalent refractive surgery is:
A- Myopic LASIK
B- Hyperopic LASIK
C- LASEK
D- Cataract removal with intraocular lens implantation

41- Which of the following statements regarding incisional refractive surgery is TRUE?
A- Radial incisions lead to flattening in both the meridian of the incision and 90º away.
B- Tangential incisions lead to steepening in the meridian of the incision and flattening in
the meridian 90º away (known as coupling).
C- The larger the optical zone, the greater the effect.
D- For optimum effect, the incision should be at 50% depth.

42- What is the approximate amount of corneal tissue that would be ablated in a 3 D myope
having PRK using a 6 mm optical zone and a broad-beam laser?
A- 9 μm
B- 18 μm
C- 36 μm
D- 48 μm

43- In a post-LASIK patient with diffuse lamellar keratitis, which of the following signs would
prompt immediate flap lift with irrigation as opposed to medical management?
A- Peripheral white blood cells (WBCs)
B- Central and peripheral WBCs
C- Only central WBCs with associated cell clumping in visual axis
D- All of the above

55
VIIII - INTRAOCULAR INFLAMMATION AND UVEITIS (50 q)

1- Which of the following statements about innate immunity is CORRECT?
A- It comprises recognition, processing, and effector phases.
B- It is triggered by bacterial toxins and cell debris.
C- It demonstrates specificity for each unique offending antigen.
D- It demonstrates memory, with an accelerated and more vigorous response to a
second antigenic exposure.

2- Which of the following is commonly associated with host defenses against parasitic
infections?
A- neutrophils
B- basophils
C- eosinophils
D- macrophages

3- Which of the following blood cell types is a major effector of IgE-mediated hypersensitivity?
A- neutrophils
B- basophils
C- eosinophils
D- monocytes

4- Which antibody is produced in the effector phase of the primary immune response to
antigens?
A-IgM
B- IgG
C- IgD
D- IgE

5- Which of the following disorders is associated with the least amount of inflammation?
A- Propionibacterium acnes endophthalmitis
B- sympathetic ophthalmia
C- phacoantigenic endophthalmitis
D- phacolytic glaucoma

6- HLA-B27–associated acute anterior uveitis is associated with all EXCEPT which of the
following systemic disorders?
A- Behcet’s syndrome
B- Reiter syndrome
C- psoriatic arthritis
D- ankylosing spondylitis

7- Patients with which of the following are most likely to present with granulomatous uveitis?
A- sarcoidosis
B- Behcet’s syndrome
C- juvenile rheumatoid arthritis
D- Reiter syndrome

8- Hypopyon is most likely to be seen in which of the following uveitic syndromes?
A- sarcoidosis
B- Behcet’s syndrome
C- rheumatoid arthritis
D- Reiter syndrome

56

9- Which of the following topical agents is MOST effective in controlling intraocular inflammation
in uveitis?
A- loteprednol (Lotemax)
B- fluorometholone 0.25% (FML Forte)
C- dexamethasone 0.1% (Decadron)
D- prednisolone 1% (Pred Forte, Inflammase Forte)

10- Which of the following uveitic syndromes is LEAST likely to require topical corticosteroid
management?
A- sarcoidosis
B- juvenile rheumatoid arthritis
C- Fuchs heterochromic iridocyclitis
D- Reiter syndrome

11- Periocular depot corticosteroid injections should NOT be used in which of the following
uveitic syndromes?
A- pars planitis with cystoid macular edema
B- sarcoidosis
C- toxoplasmosis
D- Reiter syndrome

12- Which of the following is most likely to be positive in a patient with acute nongranulomatous
uveitis?
A- HLA-B27
B- HLA-B51
C- HLA-B5
D- HLA-B54

13- Reiter syndrome is associated with all EXCEPT which of the following?
A- nonspecific urethritis
B- polyarthritis
C- conjunctivitis
D- ankylosing spondylitis

14- Behcet’s syndrome is associated with all EXCEPT which of the following?
A- aphthous stomatitis
B- arthritis
C- genital ulceration
D- retinal vasculitis

15- Which of the following is usually seen in childhood?
A- ankylosing spondylitis
B- Reiter syndrome
C- Vogt-Koyanagi-Harada syndrome
D- Kawasaki syndrome

16- Which of the following types of intraocular lenses is most associated with recurrent uveitis?
A- rigid, closed-loop anterior chamber intraocular lenses
B- iris plane intraocular lenses
C- sulcus-placed posterior chamber intraocular lenses
D- silicone intraocular lenses

57

17- Risk factors for the development of chronic iridocyclitis in patients with juvenile rheumatoid
arthritis include all EXCEPT which of the following?
A- female gender
B- positive rheumatoid factor (RF)
C- pauciarticular arthritis
D- circulating antinuclear antibody (ANA)

18- Which of the following is LEAST commonly seen in patients with juvenile rheumatoid
arthritis (JRA) and uveitis?
A- cataract
B- macular edema
C- glaucoma
D- band keratopathy

19- Which of the following is NOT characteristic of Fuchs heterochromic iridocyclitis?
A- unilateral uveitis
B- mild aqueous cell and flare
C- iris stromal atrophy
D- small nongranulomatous keratic precipitates in Arlt’s triangle

20- Which of the following is the most common cause of intermediate uveitis?
A- multiple sclerosis
B- idiopathic
C- Lyme disease
D- syphilis

21- Which of the following is the MAJOR cause of visual loss in pars planitis?
A- band keratopathy
B- posterior subcapsular cataract
C- epiretinal membrane
D- cystoid macular edema

22- Which of the following is LEAST characteristic of acute retinal necrosis (ARN) syndrome?
A- vitritis
B- occlusive arteriolitis
C- extensive retinal hemorrhages
D- multifocal yellow-white peripheral retinitis

23- Which of the following is commonly associated with an immunocompromised state?
A- cytomegalovirus retinitis
B- herpes simplex keratouveitis
C- acute retinal necrosis syndrome
D- ocular histoplasmosis syndrome

24- Which of the following is NOT characteristic of ocular histoplasmosis syndrome (OHS)?
A- peripapillary pigment changes
B- vitritis
C- peripheral atrophic chorioretinal (“histo”) spots
D- maculopathy

58

25- Treatment of visually threatening ocular toxoplasmosis should include which of the
following?
A- amphotericin B
B- periocular corticosteroid injection
C- pyrimethamine and sulfonamides
D- acyclovir

26- Which of the following is the BEST laboratory test for a newly acquired ocular toxoplasmosis
infection?
A- IgM antibody titer
B- IgG antibody titer
C- Toxoplasma dye test of Sabin and Feldman
D- hemagglutination test

27- Ophthalmologic signs of systemic lupus erythematosus include all but which of the
following?
A- vitreous hemorrhage
B- cotton-wool spots
C- retinal vascular occlusion
D- anterior uveitis

28- Which of the following statements is NOT correct about Wegener granulomatosis?
A- Antineutrophilic cytoplasmic antibodies (ANCA) are present.
B- There are immune complexes in small vessels.
C- The main therapeutic agents are prednisone and cyclophosphamide.
D- Sinusitis and renal disease are common.

29- Characteristics of acute posterior multifocal placoid pigment epitheliopathy (APMPPE)
include all EXCEPT which of the following?
A- It often follows a prodromal influenza-like illness.
B- It is commonly seen in the fifth or sixth decade of life.
C- Multiple cream-colored homogenous lesions are seen beneath the retina.
D- Vitreous cells and disc edema may be present.

30- Which of the following statements about syphilis and uveitis is NOT CORRECT?
A- A salt-and-pepper fundus may be seen in congenital syphilis.
B- Uveitis may be seen in secondary-stage syphilis.
C- Syphilitic uveitis cannot be cured in patients with AIDS syndrome.
D- A lumbar puncture should be performed in patients with uveitis and syphilis.

31- Which of the following laboratory tests may be negative in a patient with tertiary syphilis?
A- serum VDRL
B- serum FTA-ABS
C- serum MHA-TP
D- CSF VDRL

32- Which of the following tests is NOT used to diagnose Lyme disease?
A- ELISA for IgM and IgG
B- Lyme immunofluorescent antibody (IFA) titer
C- Western blot testing
D- culture and antibiotic sensitivity

59

33- Recommended treatment of Lyme disease includes all but which of the following?
A- penicillin
B- erythromycin
C- cephalosporins
D- tetracycline

34- Which of the following is NOT used to treat tuberculosis?
A- ivermectin
B- isoniazid
C- rifampin
D- pyrazinamide

35- Characteristics of sarcoidosis include all but which of the following?
A- elevated serum angiotensin-converting enzyme (ACE) and lysozyme levels
B- caseating granuloma on histopathology
C- pulmonary and liver disease
D- granulomatous or nongranulomatous uveitis

36- A 41-year-old Japanese man with a remote history of blunt ocular trauma in one eye but
good vision and no history of ocular surgery presents with decreased vision and severe pain
in both eyes. He has bilateral uveitis, alopecia, vitiligo, and recent cerebrovascular accident.
There is an exudative retinal detachment in one eye. Which of the following diagnoses is
most likely?
A- sarcoidosis
B- sympathetic ophthalmia
C- Vogt-Koyanagi-Harada syndrome
D- Behcet’s syndrome

37- A 67-year-old white female presents with mild uveitis with a mild vitritis and subretinal
infiltrates. The condition has been minimally responsive to topical corticosteroid treatment.
She has recently experienced weakness and confusion. Which of the following tests would
be the most important to obtain at this time?
A- gallium scan
B- Westergren sedimentation rate and C-reactive protein
C- PPD and chest x-ray
D- CT scan or MRI of the head

38- Which of the following organisms is a frequent cause of endophthalmitis after ocular trauma
but is an uncommon cause of endophthalmitis after cataract surgery or in bleb-related
endophthalmitis?
A- Staphylococcus epidermidis
B- Staphylococcus aureus
C- Haemophilus influenzae
D- Bacillus cereus

39- Which of the following is the most common cause of endogenous fungal endophthalmitis?
A- Candida
B- Aspergillus
C- Rhizopus
D- Cryptococcus

60

40- A patient with a previous mitomycin-C trabeculectomy presents with a severe bleb-related
endophthalmitis. The visual acuity previously was 20/20 and is now hand movements. Which
of the following is NOT CORRECT?
A- The visual prognosis is poor.
B- The organism in a bleb-related endophthalmitis is more likely to be Haemophilus
influenzae or a Streptococcus species than it is in a post cataract surgery
endophthalmitis.
C- Because the visual acuity is better than light perception, a vitreous tap for cultures and
injection of antibiotics should be performed.
D- Endophthalmitis may present months or years after glaucoma filtering surgery.

41- Which intravitreal antibiotic has the greatest potential for causing retinal toxicity?
A- Ceftazidime
B- vancomycin
C- gentamicin
D- penicillin

42- In contrast to the management of acute postoperative endophthalmitis after cataract
surgery, the management of P acnes chronic endophthalmitis usually requires which of the
following?
A- intravitreal antibiotics
B- removal of white plaque and capsulectomy
C- systemic antibiotics
D- periocular corticosteroids

43- Which of the following organisms usually causes the LEAST virulent endophthalmitis?
A- Staphylococcus epidermidis
B- Staphylococcus aureus
C- Streptococcus pyogenes
D- Serratia marcescens

44- Which of the following glaucoma medications should probably be avoided in a healthy
patient with uveitis, cystoid macular edema, and uncontrolled glaucoma?
A- dorzolamide (Trusopt)
B- timolol (Timoptic)
C-brimonidine (Alphagan)
D- latanoprost (Xalatan)

45- Which procedure is NOT indicated in patients with medically uncontrolled glaucoma
associated with uveitis?
A- laser trabeculoplasty
B- trabeculectomy
C- glaucoma implant (aqueous drainage device)
D- trabeculodialysis

46- The initial management of a patient with uveitis and iris bombé should include which of the
following?
A- laser iridotomy
B- surgical iridectomy
C- trabeculectomy
D- glaucoma implant

61
47- Management of cystoid macular edema in a patient with uveitis could include all EXCEPT
which of the following?
A- corticosteroids
B- nonsteroidal anti-inflammatory agents (NSAIDs)
C- acetazolamide (Diamox)
D- focal laser retinal treatment

48- Which of the following is the least frequent mode of transmission of HIV infection?
A- intravenous drug abuse
B- sexual intercourse
C- perinatal transmission
D- blood transfusion

49- Which of the following CD4 count ranges is associated with CMV retinitis?
A- 250–500 cells/mm³
B- 150–200 cells/mm³
C- 75–125 cells/mm³
D- fewer than 50 cells/mm³

50- Which of the following is the most common ocular finding in patients with AIDS?
A- herpes zoster
B- HIV retinopathy
C- Candida
D- toxoplasmosis

62
X – GLAUCOMA (34 q)

1- In the physiology of aqueous humor formation, the process that is energy-dependent and
independent of the intraocular pressure is
A- Ultrafiltration
B- Active transport
C- Simple diffusion
D- Bulk flow

2- The prevalence of glaucoma is
A- Equal in blacks and whites
B- Two times more common in whites than in blacks
C- Eight to ten times more common in whites than in blacks
D- Three to six times higher in blacks than in whites
E- Two times higher in blacks than in whites

3- The blood–aqueous barrier function of the ciliary body is
A- Related to tight junctions within the pigmented epithelium as a result of cellular
membrane permeability restrictions
B- Related to tight junctions between the apexes of non-pigmented epithelial cells
C- Related to the integrity and lack of permeability of the vascular elements within the
ciliary body
D- The reason the aqueous and blood concentration of ascorbate are the same

4- The following statements about static threshold perimetry are all TRUE EXCEPT:
A- It allows high-quality perimetry to be performed by perimetrists who have not
undergone extensive training.
B- It is easy for the patient.
C- The results depend on the size and luminance of the stimulus.
D- The data can be summarized and analyzed using statistical programs.
E- The results need to be interpreted in light of the patient’s overall clinical picture.

6- The following statements all characterize normal vessels in the angle EXCEPT:
A- They frequently branch out over the trabecular meshwork in the interpalpebral space.
B- They are radial.
C- They may run circumferentially around the periphery and be visible in only part of the
angle.
D- They originate from the iris, ciliary body, and anterior ciliary arteries.

7- Elevated IOP (≥ 22 mm Hg) is
A- Commonly caused by alcohol consumption, particularly in individuals who are not
regular consumers of alcohol
B- Caused in part by defective auto-regulation of the peripapillary capillaries
C- A risk factor for the development of glaucoma
D- All of the above

8- The Goldmann applanation tonometer
A- Is of little value in individuals with >5 D of corneal astigmatism
B- Is not affected by alteration in scleral rigidity or by corneal thickness
C- Displaces approximately 50 μl of fluid from the anterior chamber
D- Is the most valid and reliable of currently available applanation devices
E- All of the above

63


9- All of the following statements regarding Goldmann applanation tonometry are TRUE
EXCEPT:
A- The diameter of the applanated area is 3.06 mm.
B- The tear film creates surface tension that increases the force of applanation.
C- The cornea tends to resist deformation, which tends to balance out the surface
tension effect of the tear film.
D- The IOP tends to be overestimated in eyes with low scleral rigidity.

10- Which of the following visual field defects is most characteristic of glaucoma?
A- Bitemporal hemianopia
B- Paracentral scotoma
C- Central scotoma
D- Superior quadrant anopia
E- Inferior quadrant anopia

11- Each of the following conditions may produce nerve fiber bundle visual field defects similar
to those seen in glaucoma EXCEPT:
A- Chronic papilledema
B- Optic disc drusen
C- AION
D- Occipital infarction
E- Branch retinal artery occlusion

12- Glaucoma with statistically normal intraocular pressure
A- Is relatively rare
B- Should be routinely investigated with CT scans for possibility of tumor
C- Can be differentiated from glaucoma with elevated IOP by visual fields and optic nerve
characteristics
D- None of the above

13- The following are histologic changes in glaucoma EXCEPT:
A- Posterior bowing of the lamina cribrosa
B- Thinning of the retinal nerve fiber layer
C- Loss of the outer nuclear layer of the retina
D- Loss of ganglion cells in the retina
E- Peripapillary atrophy of the choroid

14- Secondary angle closure with pupillary block is the usual mechanism for glaucoma in each
of the following EXCEPT:
A- An intumescent lens
B- Iris neovascularization
C- Microspherophakia
D- Uveitis
E- Ectopia lentis

15- The iridocorneal endothelial (ICE) syndromes include all of the following EXCEPT:
A- Chandler syndrome
B- Axenfeld-Rieger syndrome
C- Iris nevus syndrome
D- Essential iris atrophy

64
16- A patient with nanophthalmos presents with angle-closure glaucoma. Your attempts at laser
peripheral iridectomy are unsuccessful. Your next procedure should be
A- Trabeculectomy
B- Combined phacoemulsification and cataract extraction
C- Surgical peripheral iridectomy
D- Argon laser gonioplasty or iridoplasty

17- Ciliary block glaucoma, or malignant glaucoma, is associated with all of the following
EXCEPT:
A- Responds to aqueous suppressant and hyperosmotic medical management in
approximately 50% of cases
B- Secondary posterior misdirection of aqueous into the vitreous cavity occurs
C- Occurs only after incisional surgery and never following laser treatment
D- Occurs most commonly in eyes with a history of angle-closure glaucoma
E- May occur in aphakic or pseudophakic eyes

18- Which of the following causes of developmental glaucoma does NOT involve
trabeculodysgenesis as a part of its pathophysiology?
A- Sturge-Weber
B- Homocystinuria
C- Aniridia
D- Peter’s anomaly

19- Which of the following statements about primary congenital glaucoma is FALSE?
A- 80% of cases are diagnosed by 1 year of age.
B- 70%–75% of cases are bilateral.
C- Most inherited cases are autosomal dominant.
D- 65% of patients are male.

20- Which of the following beta blockers demonstrates the relative selectivity in the manner
described?
A- Betaxolol: relatively selective for beta2 receptors
B- Timolol: relatively selective for beta2 receptors
C- Levobunolol: relatively selective for beta2 receptors
D- Betaxolol: relatively selective for beta2 receptors
E- Levobunolol: relatively selective for beta2 receptors

21- The following statements concerning pilocarpine are TRUE EXCEPT:
A- By relaxing tension on the zonular fibers, it may cause narrowing of the anterior
chamber.
B- It is a direct cholinergic agonist.
C- It reduces IOP by increasing aqueous outflow.
D- It inhibits acetylcholinesterase.
E- It is relatively contraindicated in the treatment of uveitic glaucoma.

22- The following statements regarding the topical carbonic anhydrase inhibitor dorzolamide are
TRUE EXCEPT:
A- It lowers IOP by decreasing aqueous production.
B- Difficulties with ocular penetration prolonged the development of this compound.
C- It has the potential for side effects similar to those of systemic carbonic anhydrase
inhibitors, though less frequent.
D- It has been shown to be clinically effective for once-daily administration.
E- It may cause Stevens-Johnson syndrome.

65
23- The agent most likely to be associated with follicular conjunctivitis is
A- Carbachol
B- Betaxolol
C- Brimonidine
D- Dorzolamide
D- Timolol

24- When performing argon laser trabeculoplasty, the most appropriate laser settings are
A- 200-μm spot size, 0.2-sec duration, 600-mW power
B- 50-μm spot size, 0.1-sec duration, 800-mW power
C- 100-μm spot size, 0.5-sec duration, 1500-mW power
D- 50-μm spot size, 0.1-sec duration, 1500-mW power
E- 100-μm spot size, 0.1-sec duration, 800-mW power

25- The following early complications have been associated with filtering surgery EXCEPT:
A- Corneal vascularization
B- Endophthalmitis
C- Hypotony maculopathy
D- Cystoid macular edema
E- Loss of vision

26- Indications for combined cataract and filtering surgery include all of the following EXCEPT:
A- Uncontrolled glaucoma when vision is significantly impaired by cataract
B- Visually significant cataract in a glaucoma patient with advanced field loss
C- Visually significant glaucoma in a patient poorly tolerant of medical therapy
D- Visually significant cataract in a patient requiring multiple medications to control IOP
E- Visually significant cataract and very high IOP from acute pupillary block

27- Glaucoma drainage device implantation may be indicated when IOP is uncontrolled in all of
the following EXCEPT:
A- Failed trabeculectomy with antifibrotics
B- Blind painful eye with neovascular glaucoma
C- Active uveitis
D- Inadequate limbal conjunctiva
E- Following scleral buckle and vitrectomy

28- The secondary angle-closure glaucoma in which the PAS extend anterior to Schwalbe’s line
is
A- Axenfeld-Rieger syndrome
B- Neovascular glaucoma
C- ICE syndrome
D- Fuchs heterochromic iridocyclitis

29- The form of primary angle-closure glaucoma that does NOT have pupillary block as its only
mechanism for angle closure is
A- Aniridia
B- Acute angle-closure glaucoma
C- Subacute angle-closure glaucoma
D- Plateau iris

66
30- The condition in which iris neovascularization is NOT associated with peripheral anterior
synechiae and secondary angle closure is
A- Fuchs heterochromic iridocyclitis
B- Ocular ischemic syndrome
C- Central retinal vein occlusion
D- Chronic retinal detachment

31- The prevalence of primary open-angle glaucoma is higher in all of the following populations
EXCEPT:
A- Blacks versus whites
B- Patients with elevated intraocular pressure
C- Seventy-year-old women versus 80-year-old men
D- Patients with a primary relative with known primary open-angle glaucoma

32- An 80-year-old white male presents with poor vision in his right eye with sudden onset of
pain and conjunctival hyperemia. The examination reveals an IOP of 45 with a prominent cell
and flare reaction without keratic precipitates, a dense cataract, and an open anterior
chamber angle. The most likely diagnosis is
A- Phacolytic glaucoma
B- Phacoanaphylaxis
C- ICE syndrome
D- Fuchs heterochromic iridocyclitis

33- The following ocular side effects have all been associated with the use of latanoprost
EXCEPT:
A- Hypertrichosis
B- Increased iris pigmentation
C- Cystoid macular edema
D- Decreased uveoscleral outflow
E- Periocular pigmentation

34- The following are all associated with the use of brimonidine EXCEPT:
A- Increased uveoscleral outflow
B- Non-selective alpha agonist
C- Follicular conjunctivitis
D- Additivity to beta blocker
E- IOP lowering equivalent to timolol

67
XI - LENS AND CATARACT (33 q)

1- The normal, aging human crystalline lens
A- Develops an increasingly curved shape, resulting in more refractive power.
B- Develops an increasingly flatter shape, resulting in less refractive power.
C- Undergoes an increase in index of refraction as a result of decreasing presence of
insoluble protein particles.
D- Undergoes a decrease in index of refraction as a result of decreasing presence of
insoluble protein particles.
E- A and C are correct.
F- A and D are correct.

2- The following is TRUE of the lens epithelium:
A- It is a single layer of cells.
B- Newly formed epithelial cells migrate toward the anterior capsule where they
differentiate into fibers.
C- As the cells elongate and increase in size and mass, they lose organelles, making
them less dependent on glycolysis.
D- A and B are correct.
E- A, B, and C are correct.

3- With accommodation
A- Most of the change in lens shape occurs in the central anterior capsule.
B- Most of the change in lens shape occurs in the peripheral anterior capsule.
C- Most of the change in lens shape occurs in the central posterior capsule.
D- Most of the change in lens shape occurs in the peripheral posterior capsule.
E- Because of the even distribution of zonular fibers, lens shape changes are equally
distributed throughout both anterior and posterior surfaces.

4- When the ciliary muscle contracts
A- The diameter of the muscle ring is reduced, thereby increasing tension on the zonular
fibers, which allows the lens to become more spherical.
B- The diameter of the muscle ring is increased, thereby increasing tension on the
zonular fibers, which allows the lens to become more spherical.
C- The diameter of the muscle ring is reduced, thereby relaxing tension on the zonular
fibers, which allows the lens to become more spherical.
D- The diameter of the muscle ring is increased, thereby relaxing tension on the zonular
fibers, which allows the lens to become more spherical.

5- Which of the following systemic diseases is NOT associated with ectopia lentis?
A- Homocystinuria
B- Ehlers-Danlos syndrome
C- Marfan syndrome
D- Myotonic dystrophy
E- Sulfite oxidase deficiency

6- The epidemiology of cataracts suggests that
A- They are more prevalent in those under 65 years of age.
B- They are more prevalent in women.
C- They occur only as a consequence of age.
D- They rarely lead to blindness.

68

7- With regard to the embryology of the lens, which of the following statements is FALSE?
A- The anterior lens capsule is the true basement membrane of the lens epithelial cells.
B- The lens is derived from neuroectoderm.
C- Lens sutures arise from interdigitation of lens fibers.
D- The tunica vasculosa lentis regresses before birth.

8- Which of the following statements about functional visual impairment caused by cataracts is
FALSE?
A- “Second sight” is caused by lenticular myopia and improves near vision without
correction.
B- Monocular diplopia caused by cataract cannot be corrected by spectacles.
C- Mild posterior subcapsular cataracts never cause visual symptoms.
D- Cataract can cause greater impairment in contrast sensitivity than in Snellen acuity.

9- Which of the following statements about ectopia lentis in Marfan’s syndrome is FALSE?
A- It can cause monocular diplopia.
B- The lens is usually subluxated in an inferior and nasal direction.
C- Anterior dislocation is associated with pupillary-block glaucoma.
D- Posterior dislocation into the vitreous cavity can occur.
E- It occurs in a majority of patients with Marfan’s syndrome.

10- Which of the following is NOT a remnant of the tunica vasculosa lentis?
A- Lens coloboma
B- Epicapsular star
C- Mittendorf dot
D- Pupillary strand

11- Cataracts may be medically managed by all the following EXCEPT
A- Taking an aspirin twice a day.
B- Having a careful refraction and updating spectacle correction.
C- Using dilating drops.
D- Increasing ambient illumination and spectacle add.

12- Which of the following statements concerning cataracts resulting from exposure to ionizing
radiation are TRUE?
1. A latent period of up to 20 years may pass before the cataract is clinically apparent.
2. Older patients are more vulnerable than younger patients.
3. The initial clinical sign of radiation-induced cataract is nuclear sclerosis.
A- Only statement 1 is true.
B- Only statement 2 is true.
C- Only statement 3 is true.
D- Statements 1 and 2 are true.
E- Statements 1, 2, and 3 are true.
F- None of the statements is true.

13- Which type of congenital cataract is LEAST LIKELY to be visually significant?
A- Anterior polar
B- Nuclear
C- Lamellar
D- Membranous

69

14- In preparation for cataract surgery, each of the following conditions in the ocular history is
important EXCEPT
A- History of patching one eye as a child.
B- Family history of cataracts.
C- Previous ocular trauma.
D- Refractive surgery 4 years ago.
E- Previous iritis in the involved eye that has been quiet for 1 year.

15- Which of the following preoperative measures has been proven MOST effective in reducing
the risk of endophthalmitis?
A- Administering oral amoxicillin beginning 3 days before surgery
B- Prescribing topical antibiotics for 2 weeks following surgery
C- Decreasing the duration of surgery
D- Administering topical 5% povidone-iodine solution at the time of surgery
E- Injecting vancomycin into the infusion/irrigating solution

16- The advantages of anterior capsulotomy by capsulorrhexis include
A- Allows for better fixation of the nucleus during phacoemulsification
B- Allows for better centration and stability of the IOL
C- Helps ensure IOL placement in the bag
D- More resistant to radial tears
E- All of the above

17- As the cataract incision is moved more posteriorly
A- The incidence of postoperative against-the-rule astigmatism increases.
B- The risk of bleeding in the wound decreases.
C- There is less induced with-the-rule astigmatism in the initial postoperative period.
D- The visual recovery is delayed.

18- Appropriate management of severe retrobulbar hemorrhage includes all of the following
EXCEPT
A- Proceeding with surgery if the red reflex is maintained.
B- Prompt and firm direct pressure upon the globe.
C- Observing the optic nerve and fundus with an indirect ophthalmoscope.
D- Administering carbonic anhydrase inhibitors or mannitol intravenously to reduce
intraocular pressure.
E- Performing a lateral canthotomy if proptosis, increased intraocular pressure, and tight
eyelids persist after other measures have been undertaken to relieve orbital swelling.

19- If the posterior capsule ruptures and nuclear material falls back into the vitreous during
phacoemulsification, the surgeon should
A- Immediately terminate the case.
B- Send immediately for a vitreoretinal surgeon.
C- Make every possible attempt to retrieve the lost piece of nucleus.
D- Remove any remaining nuclear and cortical material from the posterior chamber and
perform a vitrectomy.
E- Never consider placement of an IOL in that case.

20- The advantages of topical anesthesia for cataract surgery include
A- Eliminating the risk of ocular perforation.
B- Eliminating the risk of postoperative ptosis.
C- Eliminating the need for intravenous sedation.
D- Eliminating the risk of postoperative diplopia.

70
E- Eliminating the risk of suprachoroidal hemorrhage.
21- Which of the following statements about phacoemulsification incision burns is TRUE?
A- They are less of a problem with clear corneal incisions.
B- They occur only as a result of inadequate fluid inflow.
C- Heat is generated during phacoemulsification surgery by frictional forces resulting
from the vibration of the phaco tip.
D- The use of chilled BSS has solved the problem.

22- If ciliary block glaucoma is suspected as causing a shallow anterior chamber after cataract
surgery, all of the following maneuvers may be useful EXCEPT
A- Miotic drops such as pilocarpine to constrict the pupil, deepen the anterior chamber,
and open up the trabecular meshwork.
B- Aqueous suppressants such as beta blockers and carbonic anhydrase inhibitors to
lower the intraocular pressure.
C- Nd:YAG laser disruption of the anterior hyaloid face.
D- Mechanical vitrectomy to decompress the vitreous and disrupt the anterior hyaloid
face.

23- Consider the following statements about postoperative endophthalmitis. Choose the ones
that are correct.
1. The most common organisms grown from vitreous cultures are gram-negative rods.
2. Pars plana vitrectomy is of greater benefit than vitreous tap in acute cases of
endophthalmitis occurring within the first 3 days of surgery.
3. Intravenous antibiotics are of benefit for the first 48 hours.
4. Intravitreal corticosteroids should never be administered until culture results are available.
A- 1 and 3 above are true.
B- 2 and 4 above are true.
C- Only 2 is true.
D- Only 4 is true.
E- None of the above.

24- If the capsulorrhexis tear starts to extend too far peripherally, the following maneuver(s) may
be utilized:
A- Check for positive vitreous pressure and try to relieve any external pressure on the
globe.
B- Refill the anterior chamber with viscoelastic.
C- Insert a second instrument through the paracentesis site to press posteriorly on the
lens.
D- Use the bent cystotome to try to redirect the tear centrally.
E- All of the above.

25- During phacoemulsification, high vacuum is desirable for all of the following reasons
EXCEPT:
A- High vacuum allows for lower levels of phaco power.
B- High vacuum allows for easier emulsification of hard nuclei.
C- High vacuum helps to reduce lens chatter.
D- High vacuum makes nuclear sculpting easier and safer.

26- Postoperative cystoid macular edema is
A- More common after ECCE than after ICCE.
B- Always associated with visually symptomatic patients.
C- Almost always a result of the IOL.
D- Almost always treated surgically.
E- Almost always a result of increased perifoveolar capillary permeability.

71
27- Which of the following treatment approaches is MOST appropriate for a 2-month-old with
dense and diffuse bilateral cataracts?
A- Begin patching the better eye to improve acuity of the worse eye prior to cataract
surgery.
B- Perform cataract surgery on the worse eye as soon as possible. Monitor
postoperatively for signs of retinal detachment. Schedule surgery on the fellow eye at
least 4 months later to be certain there are no ocular complications from the first
surgery.
C- Arrange to perform small-incision cataract aspiration with anterior vitrectomy in one
eye to be followed, soon after, by the fellow eye.
D- After complete discussion with parents of the risk and benefits of cataract surgery,
plan bilateral ICCE as soon as possible. This will help to prevent the complications of
amblyopia.
E- If nystagmus is present, cataract surgery is not worthwhile. Visual recovery would be
negligible.

28- Evaluation of the cornea is important prior to cataract surgery. Which of the following
statements is TRUE?
A- Corneal transplant surgery should be combined with cataract extraction when guttatae
are present in order to speed visual rehabilitation.
B- Specular microscopy is the best means of determining how well the cornea will fare
following cataract surgery.
C- Normal corneal pachymetry measurements obtained in the early morning suggest that
the cornea will probably remain clear following cataract surgery.
D- Corneal pachymetry should be performed late in the day, after the cornea has had
longer exposure to the environment.
E- Other than determining lens implant power, keratometry does not have a role in the
preoperative evaluation for cataract surgery.

29- Which of the following statements about the management of cataracts in the diabetic patient
TRUE?
A- ICCE is the preferred technique because ECCE will obscure the view of the peripheral
retina.
B- IOLs should not routinely be implanted in diabetic patients; however, when indicated,
anterior chamber lenses are preferred because of the enhanced view of the peripheral
retina.
C- Nuclear cataract is the type most commonly seen in younger diabetic patients.
D- Care should be taken to protect the corneal epithelium during surgery, because it is
slow to heal in some diabetic patients, and recurrent corneal erosion may develop.
E- If posterior capsule opacification develops in a patient with a posterior chamber IOL,
Nd:YAG posterior capsulotomy should be postponed for as long as possible.
Proliferative retinopathy may develop soon after the capsulotomy is performed.

72
30- Which of the following statements is TRUE about the management of cataract associated
with ocular trauma?
A- After blunt or penetrating trauma in children, fibrin can be deposited on the anterior
lens capsule, which mimics the appearance of cataract.
B- Cataracts associated with large corneal lacerations should be removed through the
laceration to avoid making an additional corneoscleral wound.
C- If a cataract does not develop in the injured eye within 10 days of the trauma, the
patient is unlikely to develop a cataract later.
D- Phacoemulsification through a small limbal incision is the best approach to the
removal of any cataract associated with acute trauma.
E- The benefits of inserting an IOL at the time of surgery outweigh the risks when
removing a cataract during the repair of a paracentral corneal laceration.

31- A 3-year-old with a dense developmental cataract in the left eye demonstrates poor fixation
OS and a left esotropia. The right eye appears normal. Which of the following statements is
TRUE?
A- IOL implantation surgery should not be performed in children.
B- The left esotropia should be repaired surgically prior to cataract surgery.
C- Amblyopia therapy should begin prior to cataract surgery.
D- Posterior capsulotomy should not be performed at the time of surgery because of the
risk of retinal detachment.
E- Cataract surgery with IOL implantation is a reasonable approach toward visual
rehabilitation in this case.

32- A 76-year-old man complains of difficulty driving because of reduced vision. His best-
corrected visual acuity is 20/70 OD and 20/40 OS. Goldmann visual fields are constricted,
more in the OD than the OS. A moderate nuclear cataract is present OD, and a mild one is
seen OS. His IOP is 23 mm Hg OD and 18 mm Hg OS. He uses timolol 1/2% bid OD and
dorzolamide tid OD. His cup-to-disc ratio is 0.8 OD and 0.6 OS. The fundus is otherwise
normal. Which of the following statements is TRUE?
A- Cataract surgery should not be considered because of the risk of loss of fixation
postoperatively.
B- Cataract surgery combined with glaucoma filtering surgery is the only approach that
should be considered for this patient.
C- Medical glaucoma treatment should be maximized before considering cataract
surgery.
D- The visual field constriction in this case is probably caused by glaucoma.
E- The use of latanoprost after cataract surgery may increase the risk of postoperative
CME.

33- Which of the following statements about anticoagulation therapy and cataract surgery is
FALSE?
A- It is more important to maintain constant anticoagulation therapy in a patient after
mechanical heart valve implant than when atrial fibrillation is present.
B- The risk of hemorrhage in an anticoagulated patient is not reduced by administering
peribulbar rather than retrobulbar anesthetic injection.
C- The use of clear corneal cataract surgery with topical anesthetic minimizes the risk of
hemorrhage in a patient who requires anticoagulation therapy.
D- General anesthesia is the preferred technique for cataract surgery in most patients on
anticoagulation therapy.
E- Small-incision cataract surgery with peribulbar or retrobulbar anesthetic injection can
be safely performed with only a small risk of hemorrhage.

73
XII - RETINA AND VITREOUS (45 q)

1- All of these diagnostic tests are useful in evaluation of a patient with a retained magnetic
intraocular foreign body EXCEPT:
A- Indirect ophthalmoscopy
B- Computed tomography
C- Electrophysiology
D- Magnetic resonance imaging
E- Echography

2- Which vitreoretinal abnormality is most likely to lead to retinal detachment?
A- Lattice with atrophic holes
B- Operculated holes
C- Acute horseshoe retinal tear
D- Meridional complex
E- Acute posterior vitreous detachment

3- In phakic asymptomatic patients, which of the following types of retinal break is almost
always treated, while the others are rarely treated?
A- Operculated tears
B- Lattice degeneration with or without hole
C- Retinal dialysis
D- Atrophic holes

4- Blunt trauma may be associated with the development of all of the following EXCEPT:
A- Macular hole
B- Multiple retinal breaks at the vitreous base
C- Retinal dialysis
D- Commotio retinae (Berlin’s edema)
E- Branch retinal vein occlusion

5- Which of the following statements describing eyes with retained lens fragments after
phacoemulsification is INCORRECT?
A- Marked intraocular inflammation is common.
B- Secondary glaucoma is caused by lens particles and proteins obstructing the
trabecular meshwork.
C- The cumulative rate of retinal detachment is approximately 15% in these eyes during
follow-up.
D- The visual prognosis is generally poor in spite of treatment.

6- Which of the following complications of panretinal photocoagulation can be caused by
excessive treatment?
A- Angle-closure glaucoma
B- Peripheral visual field loss
C- Exudative retinal detachment
D- All of the above

74
7- Which of the following is LEAST likely to be present in an eye with a purely tractional retinal
detachment?
A- Concave surface
B- Sickle cell retinopathy
C- Smooth retinal surface
D- Extension of detachment to the midperiphery
E- Tobacco dust

8- Which of the following is most characteristic of exudative retinal detachment?
A- Shifting fluid
B- Tobacco dust
C- Fixed folds
D- Equatorial traction folds
E- Demarcation lines

9- Signs of chronic retinal detachment include all of the following EXCEPT:
A- Multiple demarcation lines
B- Subretinal deposits
C- Red vitreous hemorrhage
D- Intraretinal cysts

10- Based on ETDRS reports, which of the following statements regarding the use of aspirin is
INCORRECT?
A- No effect on visual acuity
B- No effect on progression of retinopathy
C- No effect on rates of vitreous hemorrhage
D- No effect on rates of progression to high-risk PDR
E- Significantly increases the rate of vitrectomy for non-clearing vitreous hemorrhage

11- Based on DCCT reports, which of the following statements is INCORRECT?
A- Intensive glycemic control reduced the risk of new-onset diabetic retinopathy.
B- Intensive glycemic control reduced the progression of existing diabetic retinopathy.
C- Intensive glycemic control reduced the incidence of laser treatment for more severe
retinopathy.
D- The DCCT results can be applied to all insulin-dependent diabetic patients.
E- The DCCT results do not apply to non–insulin-dependent type 2 patients.

12- As defined by the Diabetic Retinopathy Study, all of the following features are significant in
determining the presence of high-risk proliferative diabetic retinopathy EXCEPT:
A- The presence of new vessels
B- Location of new vessels on or near the optic disc
C- The presence of vitreous hemorrhage
D- Localized tractional retinal detachment
E- Moderate to severe NVE

13- Regarding aldose reductase in patients with diabetes mellitus, which of the following
statements is INCORRECT?
A- Aldose reductase is the rate-limiting first enzyme in the polyol pathway.
B- Aldose reductase converts glucose into sorbitol.
C- Sorbitol accumulation in the crystalline lens may cause lens swelling and cellular
damage.
D- Aldose reductase has been detected in retinal pericytes and in renal glomeruli.
E- Aldose reductase inhibitors have been demonstrated to slow the progression of
retinopathy in a large multicenter clinical trial.

75

14- The test or sign that appears to be most helpful in predicting the development of iris
neovascularization in eyes with ischemic central retinal vein occlusion is
A- Increased intraretinal blood
B- Pupillary measurements
C- Fluorescein angiography
D- Electroretinography
E- Intraocular pressure
15- Most eyes with the ocular ischemic syndrome have an ipsilateral carotid artery stenosis of at
least
A- 25%
B- 50%
C- 70%
D- 90%
E- 100%

16- Carotid endarterectomy has been shown to reduce the incidence of disabling stroke in
symptomatic patients with
A- Ulcerated carotid plaques
B- 0%–29% carotid stenosis
C- 70%–99% carotid stenosis
D- 100% carotid stenosis
E- None of the above

17- Threshold retinopathy of prematurity consists of
A- 8 total, or 5 confluent, clock hours of extra-retinal neovascularization
B- Plus disease
C- Zone I or II disease
D- A and C
E- A, B, and C

18- Which of the following features is NOT characteristic of Coats disease?
A- Predominance of males
B- Unilaterality
C- Exudative response
D- Retinal neovascularization
E- Idiopathic

19- Complications of retinal arterial macroaneurysm include
A- Exudation
B- Retinal arterial obstruction
C- Vitreous hemorrhage
D- A and C
E- A, B, and C

20- What percent of eyes with acute central retinal artery obstruction have visible emboli in the
affected fundus?
A- 10%
B- 20%
C- 50%
D- 75%
E- 100%

76
21- Causes of central artery obstruction include all of the following EXCEPT:
A- Hemorrhage under an atherosclerotic plaque
B- Dissecting aneurysm
C- Emboli
D- Thrombi
E- Vascular rupture

22- All of the following statements about cotton-wool spots are true EXCEPT:
A- They often cause severe visual loss.
B- Diabetic retinopathy is the most common cause.
C- They often resolve within 5–7 weeks.
D- They are typically indicative of a serious underlying systemic disease.
E- They occur secondary to axoplasmic damming in areas of focal retinal ischemia.

23- Features of the ocular ischemic syndrome include all of the following EXCEPT:
A- Ocular pain
B- Prolonged recovery following exposure to bright light
C- Decreased vision
D- 5-year mortality of 40%
E- Stroke as the leading cause of death

24- A reduced and delayed cone b-wave is consistent with all of the following diagnoses
EXCEPT:
A- Retinitis pigmentosa
B- Central retinal vein occlusion
C- Cone dystrophy
D- Syphilitic chorioretinitis
E- Sectorial retinitis pigmentosa

25- Congenital stationary night blindness (with a normal fundus) is generally characterized by
A- Absent off-responses in the retina
B- A “negative-type” electroretinogram
C- Delayed regeneration of rhodopsin
D- Mizuo-Nakamura phenomenon
E- A tritan (blue-yellow) color defect

26- The most critical and constant finding in retinitis pigmentosa is
A- Dense bone speckle pigmentation in the retinal periphery
B- An abnormality in the rhodopsin gene
C- Acquired red-green color deficiency
D- A significantly reduced electroretinogram
E- Small tubular visual fields

27- In treating CNV associated with ocular histoplasmosis, the ophthalmologist can decrease
the risk of recurrent CNV by
A- Using krypton red laser rather than argon green laser
B- Using durations of 0.5 seconds
C- Covering the entire lesion with laser treatment
D- Attaining a uniform white intensity of the area of photocoagulation at least as great as
the minimal intensity standard published by the Macular Photocoagulation Study
E- C and D

77
28- An individual who is born without red-sensitive cone pigment function (protanopia) is likely
to
A- Have poor visual acuity
B- Confuse blue and yellow
C- Perceive the long-wavelength portion of the spectrum as being darker than normal
D- Manifest photophobia
E- Be hypersensitive to green

29- Primary dysfunction of the cone system (achromatopsia, rod monochromatism) is
characterized by all of the following EXCEPT:
A- Decrease in visual acuity
B- Photophobia
C- Absent color discrimination
D- Poor night vision
E- Nystagmus

30- A 52-year-old man with a complaint of poor vision for approximately 20 years is seen.
Fundus examination reveals well-delineated, atrophic macular areas with some yellowish
marginal material. No other family members are available for examination, but he states that
his maternal grandfather, his mother, and his maternal uncle also had poor vision, with onset
in the mid-30s. Work-up reveals a visual acuity of 20/200 in each eye, central scotoma with
full peripheral visual fields, a few errors with the Farnsworth Panel D-15, an electro-
oculogram ratio of 1.30, and a normal photopic and scotopic electroretinogram. The most
likely diagnosis is
A- Central areolar choroidal dystrophy
B- Old disciform macular degeneration
C- Best disease
D- Retinitis pigmentosa
E- Stargardt’s disease

31- Which of the following statements is FALSE in relation to X-linked ocular albinism?
A- Iris is translucent.
B- Carrier females cannot be detected.
C- Macromelanosomes are found in the retinal pigment epithelium.
D- Macular hypoplasia is present.
E- Nystagmus and reduced vision are features of the disorder.

32- A bull’s-eye maculopathy is MOST likely to be found in which of the following hereditary
macular dystrophies?
A- Cone–rod dystrophy
B- Vitelliform macular dystrophy
C- Central areolar dystrophy
D- Stargardt’s disease
E- Pericentral retinitis pigmentosa

33- A normal electroretinogram is usually found in all of the following diseases affecting the
retina EXCEPT:
A- Vitelliform dystrophy
B- Dominant drusen
C- Juvenile retinoschisis
D- X-linked ocular albinism
E- Pattern dystrophy

78
34- Laser photocoagulation of subfoveal choroidal neovascularization in age-related macular
degeneration has been shown to be successful in certain cases that meet which of the
following criteria?
A- Evidence of classic CNV
B- No evidence of occult CNV
C- No evidence of blood
D- Well-demarcated boundaries
E- A and D only
F- A, B, and D only

35- A 75-year-old patient with AMD and a disciform lesion in one eye and soft drusen with focal
hyperpigmentation and 20/30 vision in the fellow eye is best managed by
A- Prophylactic krypton red laser therapy to parafoveal drusen
B- Self-monitoring with an Amsler grid
C- Repeat fluorescein angiography on a tri-monthly basis to detect subretinal
neovascularization
D- Initiation of systemic vitamin E therapy
E- Protection from ultraviolet light

36- All of the following conditions predispose patients to develop a retinal detachment EXCEPT:
A- Lattice degeneration
B- Cataract extraction
C- Cobblestone degeneration
D- Cystic or vitreoretinal tufts

37- Features of degenerative retinoschisis include
A- Relative scotoma
B- Superior nasal location
C- Frequent retinal detachment
D- Two separate peripheral retinal layers identified with scleral depression

38- All of the following statements about anterior PHPV are correct EXCEPT:
A- It must be considered in the differential diagnosis of leukocoria.
B- It is unilateral in as many as 90% of cases.
C- It is associated with intraocular calcification.
D- It has recently been renamed persistent fetal vasculature (PFV).

39- Permanent visual loss after blunt trauma may be caused by
A- Choroidal rupture with or without secondary subretinal neovascularization
B- Traumatic pigmentary maculopathy
C- Retinitis sclopetaria
D- Macular hole
E- All of the above

40- All of the following are signs of shaken baby syndrome EXCEPT:
A- Intraretinal hemorrhages
B- Retinoschisis cavities
C- Lethargy, irritability, seizures, and Hypotonia
D- Optic nerve hypoplasia

79
41- Sympathetic ophthalmia
A- Occurs in approximately 1 in 1500 penetrating injuries
B- Never causes permanent loss of sight
C- May be avoided by early enucleation of unsalvageable eyes
D- Does not cause exudative detachment

42- Radiation retinopathy
A- May occur following external-beam or plaque radiotherapy for choroidal melanoma
B- May be seen after radiation therapy of sinus or orbital tumors
C- Typically presents with microaneurysms, intraretinal hemorrhages, telangiectasia, and
exudation
D- May occur after as little as 15 Gy of external-beam radiation
E- All of the above

43- Posterior vitreous detachment is associated with all of the following EXCEPT:
A- Flashes and floaters
B- Hemorrhages at the vitreous base
C- A Weiss ring
D- Tobacco dust pigment
E- Vitreous hemorrhage

44- Photodynamic therapy for eyes with neovascular AMD should be considered in cases of
A- Subfoveal occult CNV
B- Subfoveal classic CNV
C- Subfoveal predominantly classic CNV
D- A, B, and C
E- B and C

45- Clinical management decisions based on which of the following diagnostic procedures have
been shown to affect visual outcomes in randomized clinical trials of patients with CNV?
A- Fluorescein angiography
B- Indocyanine video-angiography
C- Optical coherence tomography
D- A and B